கவனிக்க: இந்த மின்னூலைத் தனிப்பட்ட வாசிப்பு, உசாத்துணைத் தேவைகளுக்கு மட்டுமே பயன்படுத்தலாம். வேறு பயன்பாடுகளுக்கு ஆசிரியரின்/பதிப்புரிமையாளரின் அனுமதி பெறப்பட வேண்டும்.
இது கூகிள் எழுத்துணரியால் தானியக்கமாக உருவாக்கப்பட்ட கோப்பு. இந்த மின்னூல் மெய்ப்புப் பார்க்கப்படவில்லை.
இந்தப் படைப்பின் நூலகப் பக்கத்தினை பார்வையிட பின்வரும் இணைப்புக்குச் செல்லவும்: Nivedini 1996.12

Page 1
WOI. 4 N Decebe
LKS LL S L S S S L L
 

ഗ്ര
, Break;
TEIg H.
Canded Ho ialaisin Arne Co
Deconstruction is Εξεταπίπτ Womens Pag SS
ES in = "Man Engliე PailiesE
Globa Forces
he 21. Centu 'பட
Hunan Right Edinaan ld E.
ኯgate Days
1996
ES TENTE

Page 2
Our Objectives
To study and research the various aspects of women's subordination in Sri Lanka in order to sensitize men and women on gender issues.
To establish a forum for women to express themselves as writers, researchers, poets, and novelists; to publish their works in Sinhala, Tamil and English.
To disseminate information relating to women and create awareness and increase consciousness on feminist issues.
To strengthen the Women's network locally and internationally.
To extend co-operation to and solidarity with other oppressed and marginalised groups in Sri Lanka (such as refugees, the unemployed and slum dwellers) with projects for rehabilitation and general upgrading of their lives.
To serve as a resource and documentation centre in Sri Lanka that will become part of the network of research and study centres on Women's Studies in the Third World.
What does Nivedini' mean?
Nivedini derives from a Sanskrit verb. It could mean either, that which is placed before you ritually and reverentially, or a carrier of knowledge with a female gender Suffix "ni" (derived from the verb vid, to know.) We use it with the second meaning.
n
(e) Women's Education and Research Centre
±
WERC

Contents....
Editorial... ... ... ... ... .
Female Headed Households in Situations of Armed Conflict - Sepali Kottegoda . . .
Cat's Eye. ... ... ... ... ... ... ... ... ... ... ... ... ... ... ... ... ... ...
Universality Vs. Cultural Relativism in Human
Rights: The Debate and a Response - Dr. Jayadeva Uyangoda .... ...
Deconstruction is Revolution if Revolution is Body Aching - Yolanda Foster... ... ... ... ...
Still Struggling..... - Shiranee Mills. ... ... ...
A Chinese Woman Breaks the Silence. ... ... ... ... ... ... ... ... ... ... ... ...
Gender Asymmetry in Labour - Shiranee Mills. ... ... ... ... ... ... ...
Through Women's Eyes-Charlotte Bunch... ... ... ... ... ... ...
Towards a Better Model for Women's Pages in Sri Lankan English dailies - Manique Karunaratne . ... ... ...
Women in the Labour Movement - Mangalika de Silva.......
Human Rights Education and Empowerment of Women - Selvy Thiruchandran . ... ... ... ... ... .
Agate Days - Eva Ranaweera . . . . . .
Statements :
1. Vivienne - An Official Assessment of the
Lanka Sama Samaja Party'* ... ... ... ... ... ... ... ... ... ... ... ... ...
2. Krishanthi's Gang Rape. ... ... ... ... ... ... ........................
3. Statement on Rev. Tissa Balasuriya's Excommunication........
Page No.
01
10
20
2I
35
43
45
48
61
66
73
84
96
104
107
I 10

Page 3
Board of Directors Ms.Anberiya Haniffa
Dr. Kumari Jayawardena MS. Bernadeen Silva Dr. Radhika Coomaraswamy Dr. Selvy Thiruchandran
Editor Selvy Thiruchandran
Typeset and Text design : Carmen Niranjala Peiris
Printed by Karunaratne and Sons Ltd.
M/2žzáz - Vol.4 No.2
A journal published by the
Women's Education and Research Centre
No.58, Dharmarama Road, Colombo 06. Sri Lanka. Tel: 590985/595296 Fax : 5963 13
ISSN 1391 - 0027
(C) Copyright 1996 by the organisation of Women's Education and Research Centre
No.58, Dharmarama Road, Colombo 06, Sri Lanka.
No part of this journal may be reproduced without permission from the publisher expect for review purposes.
Letters may be addressed to WERC, Women's Education and Research Centre No.58, Dharmarama Road, Colombo 06, Sri Lanka.

M2 Ze
We dedicate this issue of the Nivedini to the memory of Ms. Vivienne Goonewardene, a Courageous woman who worked fearlessly for the marginalised in Society.
A fiery, eloquent speaker Vivienne gave focus to the working classes and their problems and steered people's thinking and action towards the CauSeOfwOmen...........

Page 4

경R : ****

Page 5

From the Editor
The Metaphor of Growing Pains and the Continuing Challenges
in early 1982, my editorial for Voice of Women, a feminist journal, was
titled "Growing Pains'. which literally means, problems arising at the ini
tial stages of an enterprise. Fourteen years later, I pick the same theme which is proof of the fact that feminism is still struggling to grow. The enterprise of feminism is not in its initial stages as we all know. After all these years of feminism and the different phases it has undergone, we are still compelled to speak of the growing pains of feminism. Awareness programmes, Gender Sensitization programmes, the Women's Charter, the Women's Bureau, Ministry of Women’s Affairs, International Fora, UN Conventions - these abound and women's equal rights to dignity is supposedly entrenched in innumerable documents, through numerous devices and statutes. States, civil Society, international covenants, UN Bodies and agencies have devised means to achieve those demands of feminism at tremendous cost, energy and time. How deep down have they entered the consciousness of male humanity - the answer is painfully unsatisfactory.

Page 6
.á هلال.
Feminism still reels under many blows. Krishanthi's rape successively by eleven state officials and the murder of the family members and the kindly neighbour who went in search of justice have a three dimensional implication. - the coercive state apparatus and its brutality in times of war, the vulnerability of female sexuality of the enemy which has to be controlled, "spoilt" and destroyed in cultural connotations and finally the ethnic belonging of that sexuality. And now we are told through the media with authentically substantiated facts that there were many more girls who were subjugated to similar brutality. We are promised that the wrongdoers will be punished. The question that begs an answer is "when will this stop?'
One cannot help making comparisons. The judgement on the Rupan Doel Bajaj K.P.S. Gill case in India is a case in point. K.P.S. Gilla reputed Director General of Police in Punjab, a national hero who was largely responsible for eliminating terrorism in the Punjab, was charged with sexual harassment at a dinner party. The woman victim was an IAS Officer of the Punjab Cadre. Eight years after the crime, K.P.S. Gill was sentenced to rigorous imprisonment. Justice was set right finally by a Supreme Court Order. One factor which would have contributed to the flow (though staggered) of justice would have been the social and political status of the woman victim. She was powerful and had several privileges. These are de facto contributory factors which sustained the course of events. How does Krishanthi's case compare with this. Krishanthi belongs to an ethnic minority and lived in that part of the country where the state is involved in a war, to subjugate and conquer the territory from the grips of a group of youth who have resorted to armed struggle. Krishanthi's placement both territorially and ethnically has various implications if one were to take into consideration the judicial history in Sri Lanka. Civic minded common men and women have voiced their concern in this case and while the nation awaits a judgement there is scepticism all round.
The Taliban Episode, A Historical Reversal
The take-over of Kabul by the Taliban Group signifies yet another process. An anti-state organisation, a product of the unsettled refugee population Taliban was viewed as a force that would end the Civil War and bring order in Afghanistan. Developing a "pan Islamic Ideology", Taliban has singled out women in a more repressive manner, as part of its religious mission. Those men who failed to grow beards or who trimmed them are supposed to be violat

The Metaphor of Growing Pains and the Continuing Challenges
ing the Taliban edicts and are punished. Beards are considered manly symbols and though the requirement of growing beards is an imposition placed on men it does not represent a repressive order. On the contrary decrees issued on women prohibit education for girls, prohibit women from working outside homes. Violations of the dress code are viewed severely and punished. The Taliban controlled Sharia Radio announced recently that two hundred and twenty five women were punished as they did not respect the Islamic Sharia. "Women should wear the burqa and veil and respect the Sharia or face punishment. Wearing the long shapeless chador robe did not meet Islamic standards of women's dress.” - the radio announced. Burqa is a head-to-toe robe concealing the face with cloth mesh.
Like men and women the children are also issued with decrees. Marble playing, kite flying, music, dancing, television, films are also taboos. The regressive social ethos gains its legitimacy from Islam, they urge. As a result the educational system and hospital services are severely handicapped as most of the teachers and nurses are women. Nearly 26000 widows have become poorer because they cannot earn a livelihood for themselves and their dependants. Feminists and Human Rights activists are shocked and dismayed. The UN Security Council has condemned the discrimination against women but the women continue to suffer.
Men's Psychological Disorder and Violence Against Women
It has been often argued that a particular psychological disorder on the part of men single out women as targets of attack. The murders committed by men on women are explained as a particular expression of mental disorder. The Tamil films Sivappu Roja (Red Rose) and Umai Villikal (The Dumb Eyes) make a similar case. In both these films, the heroes commit a series of murders and in the latter many women are also raped. It was explained that mental disorder, brought about by the frailty of women was the cause that reduced the men to rape and murder. A news item in an Australian Newspaper deserves COment.
"I am finally set free, I am trapped inside, I can't run forever and nor can I hide' is the content of a Suicide note of Rachel Packwood. She hanged herself after being raped twice in a year. The man who raped her Costello was described by the District Court Judge, Charles

Page 7
* á الأر
Brabagan as "having the same psychological profile and limited intelligence of M. Bryant the Tasmanian Mass Murderer.”
The Tasmanian mass murderer has been made a model for comparisons. What is more intriguing is the information in the same news item -Though Costello's lawyer said his client had psychological problems, was an alcoholic, a drug addict and had an IQ of 66 which bordered on being mentally retarded, the psychiatric reports found instead that he had a mild personality disorder. (emphasis added).
It is common knowledge that when a person is angry and outraged he/ she becomes "wild' - a term usually used to refer to extreme forms of anger. It typifies a mental disorder. To explain rape as the result of mental disorder in judicial discourse is to trivialize rape. The rape of this child and her subsequent suicide as a result are matters of grave Social problems. The judicial and sociological significance of crimes are blurred in the courtroom, when claims to mental disorder are made. To what can we attribute Such claims - to male bonding? In this case the psychiatric reports point to only a mild condition of personality disorder and the terminology of personality disorder' is different from "psychological problem'- as claimed by the District Court Judge.
Personality disorders can result from sociological reasons. If personality disorder causes bestial crimes such as rape and murder, one has to analyse the whole patriarchal culture which has specific Social patterns. Psychological problems, alcoholism and drug addiction have become covers under which rape is explained. They are not reasons and causes to explain away rape. Do women experiencing psychological problems commit such crimes? This needs to be examined.
Taiwan's leading advocate for women's rights Prof. Peng Wan Ju was raped and murdered recently. Her savagely mutilated body was found near a hotel. She disappeared from the National Conference of the Democratic Progressive Party. What are the reasons for her disappearance and subsequent rape and murder. Women's activism? Oppositional politics? - Like Krishanthie she too was a victim of multiple forces. The man who is identified as the culprit may also have a mild personality disorder

The Metaphor of Growing Pains and the Continuing Challenges
Politics of Pulchritude and Feminism
When WERC issued a statement against the "Miss Tourism International Pageant 1993, held in Sri Lanka the thrust of its argument was condemning the commodification of women's bodies. However, I was not totally surprised when one man called me over the phone and wanted to know whether I was one of those plain Janes or rather whether I do "not fall under the category of a pretty damsel'. In other words he implied that the statement on the Beauty Contest was a manifestation of feminine jealousy directed against those beauties. The recent Miss World Beauty Contest in Bangalore has created mixed reactions, where even threats of self-immolation were mounted. The beauty contest had various implications for India such as Multinational invasion, liberated open economy- and those who opposed it, opposed it for various reasons: Commodification of women's bodies and sexism sent the women's movements on the march. Degrading the purity of the Indian (Hindu) soil, by portraying the halfclad woman was the moralistic objections of the Hindu Fundamentalist. (The moralistic objections are oblivious to the half-clad , seminude and nude images of women in art productions, paintings and Sculpture and India's national claims to the proud collections ranging from Kajuraho to Ajanta paintings.)
To compound the situation it is reported in the latest news letter of the Indian Association of Women's Studies (Winter 1996) that Nirmala Banerjee an economist was sceptical of the commodification theory. She is reported to have argued that if she can earn a living by selling her writing, a model who lives by selling her photographs has also to be recognised - she we can say is talking economics.
The BJP's Stand that the beauty contests are alien to Indian Culture is flawed on many counts. What is the Indian culture that is spoken of here - Has that culture given women the dignity and honour they deserve? This right wing notion of Indian womanhood being tarnished and the beauty contests which commodify the body of women are both detrimental to the interests of women for they assign subservient and inferior positions to women. Women here become the symbol of culture.
The anti - Pageant protests - the demonstrations and peaceful marches by the NGO's and BJP and the Trade Unions were met with state coercion. The protestors were mercilessly beaten up, dragged by their hair and clothes and bundled into police vans. Collaborating with the organisers of the beauty

Page 8
M2 Z('
contest is one aspect but to use coercive powers of the state to quell the democratic right to protest, is another.
The Mass Media, Beauty Contests, and advertisements have jointly created a particular notion of female beauty and has hegemonised it as the dominant image of how a beauty should look. Why should all women try to conform to this ideal which is constructed according to the dictates and needs of the male ego'? - this is another question raised.
Beauty is said to be in the eye of the beholder. The 17th century Rubens paintings, Ceres and the two nymphs portray folds and flesh, broad frames, plump buttocks, flabby thighs and rounded breasts as the ideal figure of a beauty. Ravi Varma's paintings of women also are very similar to these. In the West, beauty corresponds to some platonic visions but for the Maoris of New Zealand beauty is fat and fleshy. The Pandungs consider droopy breasts as fine.
These ideal visions and the Taliban demand of a Burqua which conceals the face with a cloth mesh arc both directed at a common site - the site of women's bodies. Whose property are they anyway?
The national liberation movements in Sri Lanka and elsewhere have always made claims to feminist rights and class and caste analysis also have formed part of the struggle. The content of such agendas have not been analysed by the social scientists in Sri Lanka, though there are ad hoc references to them as part of major studies of national liberation ideology. History has been replete with cases where National Liberation struggles used women during armed Struggle and when nationalism was reconstructed the State ideology of nationalism wanted women to go back to their traditional, cultural images of Womanhood. Purdah, Saree, motherhood become symbols of such nationalistic models.
The recent call of the LTTE for the women of the Eastern province to join their movement has yet another contradiction. Let me explain. The pamphlets distributed have rightly identified the politics of discrimination - Violence against women has increased. The women of Eelam have been targetted specifically. Not only Rajini and Krishanthi but there are other women too who were victimised. Dowry, casteism, Social backwardness are also problems that affect women. We have no problems with this statement so far but the latter part is open to question

The Metaphor of Growing Pains and the Continuing Challenges
“Be proud that you are a daughter of Tamil Eelam - Heroism in war is nothing new to our femininity. For you to win your rights best get ready today. Join the Women's wing of the LTTE."
These are indeed contradictions to feminist goals. Feminism reiterates regularly that sisterhood is global and presses for women's equality universally treating all women as one category within patriarchy. To say to be proud of being Eelam women Smacks of ethnic chauvinism. This may be the standard nationalistic claim of LTTE. But heroism in war is again anti feministic as feminists are for peace and are against militarism, war, destruction- in short against gun culture even to seek the rights of self-determination. Feminism teaches one to pursue democratic practice and peace as the means to an end. Participation of women in an armed struggle does not necessarily mean that the Struggle is connotative of a feminist struggle.
Vivienne's Ideology Thwarted
Vivienne passed away shortly after celebrating her 80th birthday perhaps with the satisfaction and pride that both the Left parties and the women have honoured her- the honour that she rightly deserved. On the day of her birthday WERC invited her to be the chief guest of a seminar, it organised in her honour on an appropriate subject.- Women and Labour. The papers presented were on the labouring women at FTZ, Rubber and Tea Plantations, women working abroad as housemaids and the women in the Trade Unions. Quite characteristically the papers identified that among the discriminatory and exploitative existence of the labour force there are indeed particular gender specific patterns of discrimination and marginalisation that affect women. Not that Vivi was new to these insights. She knew it all and realised a long time ago that the total liberation of society should encompass class, caste and gender as categories to be addressed and analysed. It was on women's day, March 8th that she was subjected to brutality of the coercive state where she was trampled and man-handled. Her fiery defiance is proof of her intimate involvement in Women's struggles. How the media projects Woman and trivialises women's role had become an area of interest for her of late. Vivienne wanted to theorise this media depiction and requested WERC to send her the book "Images' - which has analysed the Sri Lankan media from a feminist perspective. She

Page 9
M2 á,
rang up to say how greatly she benefitted reading the Images and how truly she agreed with its content.
She fully earned an epithet "socialist feminist" which sums upher mission and vision. Yet many of us were perturbed by the injustice done to her and her ideology when the Scroll that was presented to her at the official birthday party mentioned that she was not a feminist. Many people have problems with the word 'feminism'. It is a misunderstanding that it projects a man-hating, sheman's vision of destroying all that is "feminine'. It surprised us that revolutionary and progressive parties such as the LSSP should officially take such a stand. It was an unnecessary precaution to "save' Vivie "from being branded a feminist.” Like many of us her concerns were both class and gender. Perhaps we hope that when the official history of Vivienne's role in Sri Lanka is written there should prevail objectivity and truthfulness which would relate facts and not the imposition of one ideology or the other unsubstantiated by what she said and did. Marxism's marriage to feminism was called an unhappy marriage by Heidi Hartmann but Vivie made it a happy marriage and let us recognise it.
Papal Dismay - Pope John Paul II
And the archbishop of Canterbury Dr. George Carey it is announced celebrated Vespers at Rome's San Gregario al Ceilo Church. Among the theological matters discussed, the ordination of women was one. The Vatican says that the "Roman Catholic Church does not have the authority to ordain women because Christ chose only men as his apostles an act that showed divine Will'
Together they have said that women priests pose a hurdle and have created a serious problem between the churches.
And in Sri Lanka was excommunicated Father Tissa Balasuriya of the Liberation Theology School for having refused to sign a declaration of faith or rather a special act of faith which specifically states that Jesus has willed that women should not be ordained as priests- We wonder whether all the other priests have signed Such an act of faith Father Tissa Balasuriya has always stood for the equality of women both within the church and in society and has spoken loudly and eloquently on women's issues. His stand on the priestly

The Metaphor of Growing Pains and the Continuing Challenges
ordination of women is only a reflection of this theological conviction. It is with sadness that we realize that this same conviction that women should be ordained as priests is one of the main reasons for his excommunication.
The State, the Church, the Taliban (religious fundamentalism) the judiciary have at different levels issued statements against women. It is painful to see that the otherwise opposing segments blissfully collaborate on gender issues. The pains are growing. Male consciousness is sadly converging on a common agenda making null and void feminist inroads. The powerful positions in which these institutions and persons arc placed add to the problem.
Engels spoke of the historical defeat of woman. History has made attempts to reverse the defeat towards a process of gendered justice. Plans.Strategems, analyses, recommendations, resolutions and new laws will become part of a continuous and ongoing process. The defeat has to be reversed continuously.
Pete Seeger the folk singer continues to sing sweetly and loudly at 77, for the rights of the underprivileged for peace and justice.
He wrote
“The problems of war, militarism, terrorism (although the worst terrorists are in government); the problems of poverty amidst plenty; of discrimination and suspicion; of injustice between different ethnic groups and of discrimination against the female Sex (my sister once wrote a song called “I want to be an engineer.) If we don't solve them they are going to Solve us. These are my concerns. I know I spread myself thin. I have done so all my life." (Frontline Dec. 13 Issue p.89)
With him we too sing his famous song he wrote in 1960 when he was involved with the Civil Rights Movement “We Shall Overcome......”

Page 10
Female Headed Households in Situations of Armed Conflict:
A Note on Some Issues of Concern Sepali Kottegoda *
emale-headed households are emerging as an increasingly important con
Stituency in Sri Lankan Society and polity as we move towards the 21st
century. At the national level, the proportion of female headed households has been noted to have increased from 16% in 1981 to 19% in 1992 to 21% in 1994. Social and economic developments in Sri Lanka in the past two decades are seen to have contributed to the increased burdens of poor women whether in paid or unpaid work. It has been found that the number of households in poverty increased over the last two decades particularly in the rural areas where women form the bulk of unpaid family labour in agricultural activities. Studies have shown, for example, that in the Dry Zone villages the poorer the households, the greater the relative contribution of women to family income. While the number of economically "active" women has grown over this period, they are found to be mainly concentrated in the category of employee rather than employer. Wages paid to women are lower than men: on average nationally, women earn only 25% of the incomes earned by men".
* Ms. Kottegoda is a researcher and is presently teaching women's studies at the Colombo University in the M.A. prograntine. She is attached to the Media Collective in Sri Lanka.
10

Female Headed Households in Situations
Combined with these general features of the situation of women within the Socio-economic framework of the country, the increase in the number of female-headed households is seen to have been the result of the political upheavals Sri Lanka has experienced in the form of social strife in the South of the country during the late 1980s and early 1990s and, continues to experience in the on-going war in the North and East. In effect, for over two decades, the population of the country has been subjected to large scale displacement from village to refugee camp or from village to other villages and, death and disappearances of family members.
According to the Ministry of Shipping, Ports, Rehabilitation and Reconstruction, by the end of 1996, a total of 785, 187 persons have been displaced of which 75,377 are children under 5 years of age. (See Map). It has to be noted that most official statistics are not gender desegregated and the preVailing situation of conflict in the North and the East has impeded the carrying out of a census of the populations in these areas for several decades. Some relief agencies such as Oxfam and FORUT are reportedly initiating moves to gather gender desegregated data of the refugee populations in the North and East. Given that in the North and East as well as the South, military action has been primarily undertaken by males (be it in the Government armed forces or militant groups) there is a general perception that approximately 60-70% of the displaced population in the country is female. Micro studies also indicate that there is a significant number of young widows amongst the female heads of household which have emerged in recent years. Official information on female-headed households still continue to describe them as households with widows', 'vulnerable women', destitute women', and unsupported women', thus generally overlooking the important distinction that what is being noted is in fact a relatively new and distinct social phenomenon with far reaching implications for our Society and economy.
Micro studies carried out in the North, East and the South indicate that female-headed households are a significant proportion of this population. A Study carried out in the South eastern district of Moneragala, which witnessed much of the violence that took place in the South of the country in the late 1980s, found that a large number of men had been killed or had 'disappeared during this period and the population now comprised 59% females and 41%
11

Page 11
males (Wijayatilake, 1994) While the overall average population composition rate in the country is female to 1 male, the implication of this study is that these female headed households are a relatively new phenomenon.
Recognising Female-Headed Households
The patriarchal social norms which prevail in Sri Lanka places higher Social and economic value on men and perceive males as having more authority in decision-making both within and outside the household. Consequently, the manifestation of these perceptions can be seen in the official (although technically unwritten) recognition of males as household heads particularly in areas such as census data collection, access to loans, access to poverty alleviation benefits and access to land or housing ownership.
The perception of male authority also pervades domestic relations in which both women and men would first look to the adult males in the household for a decision, particularly in relation to the public sphere. The emergence of the phenomenon of female-headed households, which are in effect households with no adult male/s present, thus implies a need for a serious reappraisal of household composition in official policy approaches to households in general.
A pioneering study of households affected by the on-going war in the North-East found that there are several reasons for the emergence of femaleheaded households within the past two decades (Samuel , 1994). Samuel observes that women become heads of households due to:
水 The death of a spouse killed in action as a combatant, killed by either of the forces engaged in combat; killed as a result of being caught in the cross fire, shelling, bombing, strafing or in explosions. In most such instances, the body of the victim is available to the next of kin for burial or mourning rituals to be carried out.
来 The death of a spouse subsequent to arrest or abduction by either of the
combatant forces where the death can be verified through direct admission, eye witness accounts etc.
12

Female Headed Households in Situations
: A spouse 'disappears' with no eye witness present.
ck A spouse has fled to escape harassment, arrest, detention, forced conscription or death, his whereabouts known or unknown. spouse has abandoned his family to join a militant organisation.
Families have been separated in the course of flight and a spouse cannot be located.
Although the types of situations cited above are by no means exhaustive, studies of the impact of armed conflict both in the North East and the South indicate that the situations describcd above are applicable across the country. The importance of noting the different situations through which women become heads of households is that it has implications for both the personal and public spheres.
Implications for the Survival of Female-Headed Households
In identifying the needs and concerns of female-headed households in the context of armed conflict, there are several key areas that should be considered. These areas can be categorised as broadly falling into (i) the emotional and personal needs of these women and their households, and, (ii) the recognition of their needs and concerns at the public level. At the same time, there will be instances where the public and personal situations may merge.
Personal Needs and Concerns
At the personal level, the loss of a spouse and /or sons is, in itself, an event of great sadness in any circumstances, and a sudden loss additionally causes shock. The loss of such an individual/s in the context of a Socio-cultural milieu which sets high value on men and enforces a dependence on men, has a particular impact on women who suddenly find themselves in situations of being the main decision-maker and economic provider for their families. This is often a role for which they are psychologically ill prepared. Often the circumstances in which these changes occur do not leave women sufficient social space or time to articulate their grief or concerns which in turn impact on their ability to build up their own strengths and capacities. This has been found to be
13

Page 12
'2 هلال
common in the case of women who have been displaced and are in refugee camps (Vivekanandan, 1996). Such situations have also been found in studies that have been conducted in southern villages which had been subject to armed conflict (Wijayatilake, 1994, 14).
Trauma: Where the death of such persons has been witnessed by other family members inevitably brings even more grief through the vivid memories of the suffering which the victim suffered. The psychological needs of female heads of households remains an area to which there is little attention yet paid in most government-led assistance programmes, whether in the war situation which prevails in the north and the east or in the case of the South. In recent years, a handful of NGOs have taken the initiative to provide Some facility for women to articulate their personal feelings as a means of coping with trauma (De Silva, 1996) The Centre for Family Services, for example, runs a programme which integrates the concept of helping women through their trauma and their children's traumas with counselling to enable the acquisition of marketable skills for the economic improvement of the households (Samuel, 1994, 18).
Sexual pressures: Studies have also found that women who have become heads of households due to death, disappearance or separation from their spouses in situations of armed conflict, are also subject to Social and sexual harassment from men of their own community or village or any of the armed forces/groups. The social norms and behavior which prevail in the country result in the perception of such lone women as being available for the sexual gratification of
Ոhem.
Companionship needs. In this regard it is also important to note that women who have been bereft of an authoritative and socially accepted male companion may seek various forms of companionship for purposes of both emotional and physical security. In some instances, men may move into the woman's household for short periods of time (sometimes in the event of the woman receiving any economic benefits through state assistance programmes) and subsequently move out (Wijayatilake, 1994, 15). In such situations it is important that organisations and the state agencies which work with and/or formulate programmes to assist female-headed households, in these circumstances recognise and address women's needs for companionship. They should work towards forming support networks which would empower these women and mitigate their vulnerability in Such situations.
14

Female Headed Households in Situations
Need for Income : Many of the women who have become heads of households in this context have been largely dependent on the income of their spouses for the survival of their households. With the sudden death or disappearance of the men, these women are overnight compelled to take on the added burden of being the main income contributor to their households. Most of these women have little marketable skills and knowledge on how to embark in income earning activities and thus often face immense hardship in finding the resources to maintain their families. Those who can have been found to undertake food preparation and processing at home for sale, others who can find help for childcare become casual agricultural labourers while still others end up mainly dependent on whatever State assistance they can obtain. Hence, enabling these female headed households access to marketable skills and income earning work which they can undertake, within their specific abilities as well as social and cultural contexts, is of paramount importance in State policy regarding the welfare of these families.
State support: Studies also reveal that insufficient information regarding the State benefits to which such affected households are entitled, often results in the female household heads being unable to utilise these benefits appropriately (Samuel, 1994: De Silva, 1996).
Public Concerns and Needs
Problems of official recognition. In the public sphere, the official recognition of the bereaved family's access to any beneficiary programmes initiated by the State requires a formal record as to whether the death of the spouse was witnessed or admitted to by the perpetrators of the deed. Where there has been no such record, and the spouse has disappeared in any of the other situations cited above the State policy in this regard does not allow for the newly formed female-headed household to benefit from Such programmes. This has serious implications for the survival of female-headed households particularly in those areas most affected by armed conflict.
Currently, compensation is paid to persons who are/were killed during armed conflict by militants, be it in the North East or the South. If the dead person is pronounced to a member of a militant group, the family is not recognised as being entitled to compensation. There are instances reported when
15

Page 13
/മല്ലേ
family members go to claim the body, where one has been found as a victim who had been killed by the security forces by mistake or by being caught in cross fire, they are pressured to accept the body as that of a member of a militant group. This results in the family being unable to claim compensation.
The Government that came into power in 1994 has attempted to mitigate some of the problems, that arise as a result of the lack of death certificates of spouses who are unaccounted for as a result of armed conflict, through the setting up of a Commission on the Disappeared through which compensation could be claimed by the victims' families. Those families/women who had lost a member of the household during the situation of armed conflict particularly within the last decade are encouraged to send in details of their disappearance for consideration to the Commission which sits regularly in different parts of the country. While these hearings and Subsequent decisions of the Commission have assisted some families, there are many women in the North-east as well as the South, especially those whose spouses or sons disappeared, who still find it difficult to accept the fact that they may be dead and wait in hope that they would return. Thus they are reluctant to Submit their cases to the Commission as the act in itself is seen as their acceptance of the death of their loved ones.
Lack of sensitivity/consultation by official agencies: An important issue for female heads of households in such situations is the attention paid to their concerns and capabilities by the authorities who administer public services and institutions. Women living in refugee camps often find that they are neither consulted nor included in most decisions regarding their living conditions, their physical mobility or location of resettlement. Such lapses are significant and has overall negative implications for the success of these activities and programmes (Vivekanandan, 1996). For example, it has been found that despite women articulating to some degree their gendered roles in Society, and repeatedly stressing the need for access to clean toilet facilities and cooking facilities, these needs tend to be disregarded in the course operations of moving large populations from one geographical location to another and Settling them in places chosen by the authorities (Vivekanandan, 1996).
Social acceptance: Female heads of households which have emerged in areas of continuing armed conflict and/or in areas which have been Subject to armed conflict in the past often find that the Social framework, within which these families had lived as male-headed households belonging to a community, changes with either large numbers of persons moving or being moved to
16

Female Headed Households in Situations
other geographical locations/camps or as the community or village itself is unable to accept or relate to these households as new socio-political entities. lence their right to live as equal citizens in their communities and in society in general is restricted.
Health needs: The health needs and rights of women in these situations, whether in refugee camps or elsewhere, are rarely addressed by the authorities concerned in the form of a coherent and systematic Social policy. In the context of refugee camps, primary health care facilitics are minimal and, women in these camps have often raised the issue urgent action to cater to the needs of menstruation, pregnancy and child birth - in terms of facilities and support programmes (Vivekanandan, 1996).
Concluding Observations
The concerns of female-headed households in situations of armed conflict are many. This paper has set out Some key concerns which have been identified by organisations and individuals who have been working at the ground level in areas of armed conflict. It is important to note that while recognising that female-headed households have emerged due to various circumstances albeit in situations of armed conflict, the State and NGOs need to recognise and formulate programmes to support such households and the challenges faced by these women on a daily basis.
Compilation of a comprehensive data base on the number of female headed households which have emerged in the context of armed conflict is an urgent requirement in formulating policy for this constituency of the population. The needs and concerns of female-headed households outlined above must be regarded as the genuine social rights of these women and these households. At the same time, from the point of view of social management, these concerns must be regarded as important requirements of public policy which need to be given attention as the country moves into the 21st century.
17

Page 14
/മല്ലേ
References
De Silva, G. (1996). "Gender in Disaster Mitigation: The Armed Conflict in
Southern Sri Lanka. Centre for Family Services, Colombo.
Samuel, K. 1994. War and Survival; Women Heads of Households in the East'. Options, No. 1, August. Women and Media Collective,Colombo.
Vivekanandan, S. 1996. Gender in Disaster: The Armed Conflict in Sri Lanka.
Oxfam (U.K./).
Wijayatilake, K. 1994. Case Study of Vehillini Development Centre. (Colombo).
18

Female Headed Households in Situations
Reports and Census
Department of Census and Statistics. 1995. Gender Indicators of Sri Lanka. (Colombo).
Department of Census and Statistics. 1995. Gender Indicators of Sri Lanka. (Colombo).
World Bank. 1991. Staff Appraisal Report; Sri Lanka Poverty Alleviation Project. (Washington).
UNDP, 1995. Human Development Report 1995.
Reports from the East indicate that due to the on-going conflict, an estimated 0,000 to l S,000 women have been widowed in the last 6 years, most of whom are living below the poverty line. For example, in the Eravur Pattu D.S. Division, there are 1174 registered widows most of whom are under 40 years of age.
19

Page 15
.á هلال
History of Cat's Eye
at does the phrase Cat's Eye signify? It simply means being watchful with a flare for scrutiny and a capacity for deeper analysis and interpretation. The phrase has a history in Sri Lanka. In the eighties a few progressive women got together and created a column titled "Cats Eye" in the Lanka Guardian journal. They coined this insightful title implying all the above. The column dealt with issues concerning women of Sri Lanka with a clear feminist stance. But due to reasons known to the columnists and the editor, Cat's Eye ceased to appear: It re-emerged in the Island Newspaper in 1995. A women's collective is engaged in writing this column weekly, raising very pertinent issues. The gender specific discussion that the column is engaged in, has wide popularity. Here are some excerpts.
Death to Dowry
omen object to dowry for basically two reasons. Firstly, it is a degrading
practice which makes a woman's economic status the primary factor in her
marriage not her worth as a human being. She is judged as an appendage to certain commercial goods and not as an equal partner in a marital relationship. Secondly, the practice of dowry in certain communities is so pervasive that it begins to impoverish families who are "burdened" with too many daughters. This sense of burden then begins to condition all the economic and social decisions that the members of the family make. It also begins to contribute to the community's negative attitude toward women in general. The birth of a female child is rarely welcomed as a blessing.
Today, in Sri Lanka, there is no legislation which outlaws dowry as a social practice. It is about time that women agitate to change this.
Lanka Guardian Vol.8 No. 16 December 15, 1985
Why Women mistrust the police...
s we in Sri Lanka know only too well, violence against women
who are in the custody of the state is a particularly pernicious
form of power abuse that has detrimental impact on the relationship between the state and its women citizens. Such violence resonates among all women within the state, often causing fear and apprehension towards police and military officials..........
Women's general distrust of the police stems from widespread reports of police abuse, misconduct and ill treatment of both women who are victims of violence and women in state custody.
Sunday Island February 2, 1997
20
 

Universality Vs. Cultural Relativism in Human Rights The Debate and a Response
Jayadeva Uyangoda *
Introduction
he recently re-emerged debate on the universality vs. cultural
particularism in human rights perhaps represents a third phase in the
world-wide debate in human rights in general. In the first phase, the focus of the debate was defining universally acceptable norms, definitions and standards of civil and political rights. The second phase was characterised by the expansion of the ambit of rights to include economic, social and cultural rights as well as to extend the conceptual scope of human rights to specific human communities such as the indigenous peoples, women and children. In the third phase, however, the debate has moved to a different plain -- to reexamine the philosophical, theoretical and civilizational foundations of the hitherto accepted human rights discourse itself.
* This paper was read at at Seminar on the theme "Human Rights in the North-South Dialogue" held in Colombo in November 1996, organised by the German Cultural Institute in association with the International Centre for Ethnic Studies, Colombo.
21

Page 16
'2 الألم
Until recently, except in the arguments of socialist jurisprudence and political theory, the Human Rights philosophy was based on the assumption that the rights as enunciated in the UN charter and other documents of the international community had a universal relevance, validity and application irrespective of social, cultural, economic, political or historical contexts of the states in the world. When preparatory regional meetings were held before the UN Conference on Human Rights in Vienna in 1993, a whole new debate erupted challenging that philosophical principle. Some governments as well as civil society activists of the non-Western World constituted themselves a kind of vanguard to forcefully argue that the universality principle was an essential component of the Western discourse of dominance over the non-Western world." They further argued that the non-Western societies should develop their own philosophy, principles and standards of Human Rights, quite independent of, and as against, what they viewed as the discourse of the 'dominant West'. This debate quite interestingly took a political character, invoking such political categories as West vs. the East, North vs. the South, dominant vs. the dominated, and developed VS. the developing.
In the preparatory debate mentioned above, the Asian and African countries that pressed for re-thinking and review of the universality principle saw themselves as a specific group of nations in the world community, endowed with characteristics different from the Western nations. For example, the preamble to the San Jose declaration of the LatinAmerican States reaffirmed that
[0]ur countries represent a broad grouping of nations sharing common roots within a rich cultural heritage based on a combination of various peoples, religions and races, and that our roots unite us in the search for collective solutions to present problems through friendly dialogue, peaceful co-existence and respect for pluralism and the principles of national sovereign, non-interference in the internal affairs of states and self-determination of peoples"
'. Governments that reject the universality of human rights include China, Cuba, Colombia, India, Indonesia, Iran, Iraq, Libya, Malaysia, Mexico, Myanmar, Pakistan, Singapore, Syria, Viet
van and Yenner. * I borrow this term from Chandra Muzaffar of Malaysia who is an influential thinker of the
cultural relativist school of thought and whose arguments I will examine later in this paper. . Final Declaration of the Regional Meeting for Latin America and the Caribbean of the World Conference on Human Rights, San Jose, January 1993. Cited in Cerna (1994:742).
22

Universality Vs. Cultural Relativism in Human Rights The Debate and a Response
The Bangkok Declaration of Asian States, meanwhile, stressing the universality argued for “objectivity and non-selectivity of all human rights and the need to avoid the application of double standards in the implementation of human rights and its politicalization.” It also urged (the actual term used in the Declaration is recognising) that "the promotion of human rights should be encouraged by cooperation and consensus, and not through confrontation and the imposition of incompatible values' (cited in Cerna, 1994:743).
Peter Van Ness (1995:3) summarises the main issues involved in this global debate:
i. The question of universality versus cultural or developmental universalism: whether human rights should be understood as universal principles applying to all humanity, or as values shaped essentially by the particularities of each nation.
ii. The so-called right to intervene versus the state-sovereignty against in
ternational intervention to resist human rights abuse.
iii. Competing priorities among different categories of human rights, espe
cially: (a) civil and political rights versus economic, Social, and cultural rights, and (b) individual rights versus collective or group rights.
ν. Concepts of individual rights versus citizen duties.
W. Issues having to do with the rights of indigenous peoples.
vi. Whether human rights conditions should be attached to economic and
technological assistance programs.
Global Context
The global context in which this debate has recently re-emerged has a
number of constitutive components. The seeming triumph of Western capitalism as a generalised global economic order and its political, ideological and
23

Page 17
/മ
cultural consequences led to a resurgence of intellectual nationalism in the non-Western world. The collapse of the Soviet Union and the Eastern European socialist bloc in a way placed the Western world in a position of unassailability, germinating a definite sense of renewed resistance to the West among new nationalists. When the ideologies of the Western triumphalism -- notably Samuel Huntingdon and Francis Fukuyama -- came out with visions of impending cultural conflict on a global scale and of the end of ideology as totalizing grand narratives of new globalism around the core of Western industrial civilisations, it infuriated some sections of the radical and nationalist intelligentsia so much that they developed arguments for new alliances with the states of the non-Western world to combat the new global evil. The Gulf War, once described by James Petras as the actual Third World War, in a way crystallised sentiments towards intellectual and political resistance against the Western project.
Economic globalization that has created much intellectual resistance, provides a backdrop for the debate on human rights too. As cleverly argued by Some post-modernist theorists, globalization erases local or regional specificities, arbitrarily imposes homogenisation on diversity and practices violence against local resistance. Resistance to globalization -- political, cultural, epistemological and informational -- is thus viewed as the morally right and just counterhegemonic project. The nostalgia for, and the commitment to preserve, things local and non-Western has thus become a fairly strong current in today's radical thinking.
Arguments for Cultural Relativism
In this section I will summarise the cultural relativist approach to Human rights. My Strategy is to initially summon one of its most articulate spokespersons, Chandra Muzaffar who is also the leading Asianist in the debate and then examine the implications of the general approach developed in this school of thought. At the International Conference on Rethinking Human Rights, held in Kuala Lumpur, Malaysia in December 1994, Muzaffar presented a paper entitled "From Human Rights to Human Dignity”, an abridged version of which
'. I used the term "nationalist’ here not necessarily in its ethnic sense, but in its ethno-centric and cultural relativist sense.
. Human Rights and the New World Order (1993), published by the Just World Trust, Penang, Malaysia, contains most of Muzaffar's essays on the subject.
24

Universality Vs. Cultural Relativism in Human Rights The Debate and a Response
was later published in the Bulletin of Concerned Asian Scholars in 1995. I will use that paper to illustrate his approach and arguments, although his position has been developed in a number of previous essays and public lectures.
Chandra Muzaffar's (hereafter CM) starting point is that the universality of HR argument is deeply embedded in the Western project for global domination. Therefore, the West has no moral right to be concerned with human rights in the non-West, because it has historically violated Human rights and dignity of the non-white people, has failed to uphold some fundamental rights of its own people, and continues to Support dictatorships that Suppress genuine human rights and democracy movements in its quest to perpetuate global domi
a1On.
As we know, the universality notion is fore grounded on the liberal philosophical principle of the sovereignty of the individual in the public share. Cultural relativists question this fundamental principle of the mainstream HR discourse on the argument that not all cultures, particularly the Asian ones, privilege the individual in the same way the Western tradition does. The argument then is for a cultural specific locus of the rights. CM critiques the Western notion of individualism asking the following questions (I will quote CM fully on the question of individualism to capture the rhetorical essence of his intervention):
"Is the glorified but often destructive individualism of an earlier phase in Western history now more clearly a gross, vulgar individualism that today threatens the very fabric of Western democracy? Isn't individualism of this sort a negation of the community?”
"Has the glorification and adulation of individual freedom as an end in itself reached a point where individual freedom has become the be-all and end-all of human existence? Isn't freedom in the ultimate analysis a means towards a greater good rather than an end in itself?'
The critique of the individual-centered rights discourse stems from a communitarian perspective which posits the non-Western conception of the self as fundamentally grounded on the group, the existence is perceived to have little or no meaning.
Hence CM's query: "Isn't this individual notion of freedom linked in the West to an idea of rights that is often divorced from responsibilities? Can
25

Page 18
rights be separated from responsibilities in real life?'
This critique is linked to another point, namely the 'over-emphasis by the Western and West-inspired HR community on civil and political rights, at the expense of economic, social and cultural rights. CM elsewhere states in a critique of the Amnesty International's agenda:
...Amnesty, like most Western governments, emphasises human rights practices and human rights violations which come within the ambit of political and civil liberties...Economic and social rights have received much less emphasis than they deserve. The human right to food, to clothing, to shelter, to education, to health, to employment is fundamental to the very survival of the human being... Of what use is the human rights struggle to the poverty-stricken billions of the South if it does not liberate them from hunger, from homelessness, from ignorance, from disease? Human rights interpreted mainly in terms of political and civil rights will not satisfy the quest of the poor for human dignity and social justice (Muzaffar, 1993: 12-13).
The argument presented here is plain, simple and very clear. One can only do is to attach a label to it -- the empty-belly, or human dignity perspective. But, to be fair by Muzaffar, it needs to be pointed out that he calls for "a more holistic, integrated vision of human rights' in which “life and liberty, food and freedom should go hand in hand.” This proposal for a holistic vision is not without problems and we shall examine them later. Before that a brief comment on the empty-belly perspective of human rights is necessary. Simply put, this perspective says that there is no point at all in telling a starving person that she must assert her civil and political rights.6 For the hungry, the homeless, the sick and the unemployed, the argument goes, human rights are mere abstract concepts. Much of the experience in many Asian, African and Latin American Societies, however, presents a totally contrasting picture of the relationship between belly and the rights: almost as a rule, it is often the emptybellied who become the first targets of rights violations by the state. When it comes to the denial of the rights of the empty-bellied, all state agencies tend to act with impunity.
Taking the Relativism Argument Seriously
Even one may not totally agree with the cultural relativism perspective of Human Rights, one has to grant that it is a politically evocative one which
26

Universality Vs. Cultural Relativism in Human Rights The Debate and a Response
can quite easily ruffle the Western intellectuals.7 However, one may still recognise that it raises some disturbing issues to be confronted by universalists in the developing world. Let me cite one such issue, political conditionality in development aide adopted by Western governments and multi-lateral lending institutions who dominate the global economy. The annual aid group meetings in Paris which decide the development aid policy towards individual aid recipient countries is an instance where global economic agenda of industrialised countries -- which by the way are located in both West and the East -- has to some extent co-opted the international human rights community. The cultural relativists see this as an unholy alliance designed to bully and coerce the weaker nations in the developing world and the nationalist resistance to it can sometimes legitimise in the public eye even very practices that the international HR community would abhor. The other side of the story, of course, is that the political conditionality of development aid has not always been a level-playing field when even absolutely repressive regimes are allied with more powerful nations in the West. Smarter regimes of weak nations -- perhaps Sri Lanka has been a good case in point -- learn quickly how to overcome the concerns of the international community on the requirements of political conditionality of aid.
My own response to this issue is concerned with the political-strategic utility of international mobilisation in the struggle for democratization in our Societies. And it is largely linked to the behaviour of the Sri Lankan state during the past few years. One of the most disquieting aspect of the authoritarian political practices of many states of the developing world has been the total disregard for domestic political pressure on even very grave human right violations. With occasional exceptions, regimes often behave with a conviction of impunity and the domestic human rights communities are their prime targets whenever the civil society begins to assert itself against repressive regime behaviour. In situations where democratic institutional mechanism are either weak or non-existent, ensuring of the state/regime accountability becomes extremely difficult. And indeed, regimes often behave on the assumption that accountability to domestic constituencies is totally irrelevant, unacceptable and inimical to national -- one may even say 'community' -- interests. However, the Sri
6. An Indian official attending the Colombo SAARC Summit in 1991 put across this point of view graphically when he said: "If you don't have enough food in your stomach, talking of human rights is a little abstract' (cited in Pravada, Vol. I, No.2, January 1992, p.36).
7. For such a knee-jerk response, highly enjoyable though, to the cultural relativists argument, see Johan Galtung 1992: 152-176.
27

Page 19
M% á
Lankan experience since the late eighties clearly indicates that in a context of global economic and political relations with which the state is so intimately linked, the international coni munity -- states, multi-lateral lending institutions and HR groups -- is the only constituency that a regime may view itself accountable for. At times when domestic mechanisms of checks-and-balances of State behaviour are weakened by the regime itself, the international community is perhaps the sole effective agency for checking arbitrary practices of the state, however much it may infuriate state fetishists.
In this backdrop, we in Asia need not at all feel ashamed of being members of a new global community, the international civil Society. If there is any historically meaningful internationalism today, it is the democratic internationalism to which the human rights community is committed. At least in Sri Lanka, it is the international civil Society which has been functioning, with limitations of course, as a democratic Safeguard against the arbitrary state practices, when the nationalists, neo-Stalinists nativists and cultural purists had legitimised the authoritarian state in more than one way.
Cultural Relativism as Civilizational Determinism
There is an interesting set of quasi - philosophical issues too involved in the cultural relativists critique. What is the relationship between the individual and the State in the non-Western world? Is it fundamentally different from that of the West? Is in the non-West the individual in fact secondary to the community and is the community -- nation, ethnic group, caste group, religious group etc -- the fundamental unit of human existence'? Is the non-West devoid of any concept of the self anchored on notions of sovereignty of the individual?
. A clear illustration of my argument is the immediate response that the Premadasa administration had to make when the Amnesty International made, in September 1991, thirty two recommendations to the Sri Lankan government on the improvement of the human rights situation in the country. This was the time when the domestic HR groups and oppositionist communities were under siege, after two - to three years of very grave human rights violations by the state. The government in December 1991 accepted most of the 32 recommendations. The context of this acceptance was characterised by the forthcoming Sri Lanka Aid Group meeting in Paris and the UN Commission on Human Rights in Geneva, scheduled for February, 1992. Among the matters scheduled to be discussed in Geneva was the Report of the UN Working Group on Disappearances that visited Sri Lanka in October, 1991. For details, "A Recommendations, A Speedy' Response from Sri Lanka, 'Pravada, Vol.1, No.2, January 1992, pp.23-24.
28

Universality Vs. Cultural Relativism in Human Rights The Debate and a Response
The cultural relativist responses to these questions are anchored in a world view which may be called 'civilizational determinism. It understands the world in terms of distinct civilisations -- ironically reminding us of the Huntingtonian understanding of the world as an ensemble of antagonistic civilisations. Such formulations as Asian values or African values of human rights are derived from a discourse of civilizational specificity, posited in opposition to the West. Thus, the Bangkok Declaration's preamble recognised that "while human rights are universal in nature, they must be considered in the context of a dynamic and evolving process of international norm-setting, bearing in mind the significance of national and regional particularities and various historical, cultural and religious backgrounds” (cited in Cerna, 1994: 743).
An extreme manifestation of this argument is that Human Rights, as conceptualised in the West, are irrelevant to Asian and African Societies, since the communitarian Self-understanding of people in these Societies does not carry the notion of individual Self. Rhoda Howard (1990), cites some African advocates of this position. Fasil Nahuman, for instance, argues that Africa has its own specific concept of humanism which "does not alienate the individual by seeing him as an entity all by himself, having an existence more or less independent of society.” Another spokesperson, Asmarom Legessee, makes the claim that "no aspect of Western civilization makes an African more uncomfortable than the concept of the sacrilized individuals whose private wars against Society are celebrated. "Another writer quoted by Howard completes the argumentative logic of this assertion: "The Africans assume harmony, not divergence of interests... and are more inclined to think of their obligations to other members of society rather than their claims against them."
". All cited in Howard, 1990: 162. An Asian version of the Africanist argument is articulated by Muzaffar as follows: “Unlike Western governments which see human rights as rights revolving around the individual, there is in Asia and Africa in particular a strong notion of the rights of the collectivity, the community, the nation. This has a lot to do with the colonial experience of Asian and African states. Subjected to alien, colonial rule for centuries, fighting for freedom for Whole generations of Asians and Africans came to mean fighting for the freedom of their people. There is, therefore, a concept of collective freedom and the right that go with that freedom in the historical baggage of the Asian and African. This explains why freedom for most post-colonial societies has been more than a litany of personal liberties. It is communities, of entire nations for hitman dignity and social justice’ (Muzaffar, 1993: 13-14). This paragraph is illustrative of the general tendency among cultural relativists to treat communities, ethnic groups and nations as indifferentiated, organic entities where power, domination, inequality and oppression within the so-called collectivities is totally obviated and unaccounted for. The way it fetishizes freedom struggles' of the Asians and the Africans is nicely de-constructed by the post-colonial history of many of these states which have perpetrated violence against ethnic, religious and caste minorities of their own collectivities.
29

Page 20
á لال
The centrality of the communal component of the African persons' self-making needs not be disputed even for the sake of the argument. Anthropological studies on African societies have repeatedly confirmed this. Even in the United States where modern individualistic spirit runs Supreme, community principle is not totally abscnt even among urban communities. All individual constructions of the self, whether in Asia, Africa or in the West has a communal or group dimension at varying degrees. In Africa, communal dimensions of the self may perhaps be stronger. Yet, there is a definitely uncomfortable ring around this claim for African absence of the individuality. Only a few decades ago, rulers like NKruma in Ghana and Nyrere in Tanzania used the same argument to buttress the ideology of a highly repressive and authoritarian system of governance. NKruma's ideology of African Socialism or Nyrere's concept of Ujiyama socialism celebrated the community principle in political projects only to bring disaster to entire communitarian argument: it, unlike citizen's right argument, can be easily appropriated and developed by repressive States as well as rightwing communities -- like the religious right in the US -- to deny both individual and collective rights of people. To reiterate the last point, the principle of individual-based civil and political rights can hardly be appropriated by a repressive state against its own citizenry.
In the Asian context, the same danger of the communitarian argument of rights exists, along with another paradox. Unlike the African continent in general, the Asian region represents a historically more advanced stage of capitalist development. Although Western liberal democratic institutions have not taken roots in many Asian Societies, their economy are based on a complex Web of market relations where the individual enters into transactions on a primarily capitalistic matrix. Asian capitalism cannot be seen as having re-shaped the Western capitalism in Asian societies in any communitarian fashion, except in giving rise to a repressively paternalistic regime of capital - labour relations. It thrives precisely on the individualistic market relations. The paradox of Asia then is the robust expansion of modern capitalism around industry and the market in a sizeable area of the region along an essentially Western pattern while political institutions have not. Any communitarian explanation should be utilised not to justify this paradox, but to unravel it. The query then should be: what is wrong with individual political rights when a mode of production of market economies, competitive individualism, and individual economic deciSion-making have become dominant, hegemonic and obviously unstoppable. It is precisely because the fast expanding capitalist production and exchange re
30

Universality Vs. Cultural Relativism in Human Rights The Debate and a Response
lations are tearing asunder traditional patron-client relations between the social group and the state that individual citizen-based civil and political rights become all the more relevant for Asia.
There is yet another side of this problem: in the developing world, it is not the capitalist market which atomizes the society through individuation of human relations. The state, through all its repressive, interventionist, controlling and disciplinary procedures atomises the Society as much as the market clocs. The ultimate essence of all repressive practices of the state is to treat society on the basis of individuals. The unit of human Society that suffers arbitrary arrest, torture, rape, extra judicial execution etc., is ultimately the individual. The denial of human dignity by the state is characteristically and primarily practised at the level of the individual.
The communitarian argument is fraught with another danger: its hidden assumption that non-Western Societies arc organic communities where the individual is cared for, looked after and given protective refuge by the collectivity. Except in the fertile imagination of the agrarian or tribal romanticist, this assumption can rarely be substantiated in actual Societies where internal differentiation, hierarchies and divisions negate principles of equality, justice and fairness,. Community egalitarianism in Asian and African Societies has been a perennial myth among some moderns and anti post-moderns, orientalists and Occidentalists alike. It is also a legacy of nationalist narrativization of local histories. But communities are sharply differentiated along class, caste and hierarchical lines and the advent of the modern state with its pervasive presence in society has sharpened the antagonisms within communities, the state agencies often being partisan towards the upper class, upper caste and privileged strata of communities.
This is exactly where a fundamental weakness of the cultural relativism argument of human rights reveals itself. The argument either ignores the question of the state or when it treats the State as an issue, it is often unproblematized in a manner that can be described as state fetishism. It is an argumentative fallacy to treat individual rights as disrupting communitarian principles, even if we accept the point that in Asia and Africa, the community has supremacy over the individual. What the discourse and practice of civil and political rights clocs is the protection of the individual as well as the community against the arbitrary will of the state, not against the community in its pluralistic sense. If a state claims to represent one specific community -- ethnic or tribal group in
31

Page 21
á, O الألم
the name of the nation -- as against the others, as it often happens in the contemporary world, can the dispossessed, marginalised and victimised communities seek justice by invoking community rights?
Political Justice and Social Justice as Rights.
Notwithstanding the fact that the cultural relativist argument on human rights lacks a theoretical grounding concerning politics of state-society relations, its emphasis on issues of economic and Social justice should not be dismissed off hand. What needs to be done, instead, is to work towards an inclusive rights politics -- both a discourse and practice-- so that different spheres of rights, individual and collective, are theorized in a manner that enables rightsbased intervention in politics unambiguous and meaningful. Such a theorizing effort may acknowledge, fruitfully, the historical and political circumstances within which the individual-rights based argument evolved and Subsequently the community-based rights argument emerged in Seeming opposition to civilrights discourse in the non-European world.
Community-based rights argument is essentially anchored on the primacy of economic and social rights of social collectives, in contradistinction to the rights of individual citizens. In the genealogy of the rights discourse in Societies like Ours, the approach to rights from the perspectives of social collectives has been a part of the Left wing as well as Left-nationalist politics, the origins of which go back to late colonial period of the twentieth century history. The citizen - or individual -- centered rights discourses emerged in Sri Lanka and in South Asia largely in the 1970s, in response to the authoritarian drift of the State in a context where large Scale imprisonment of citizens without trial, torture in custody, and other arbitrary practices of the state curtailing the autonomy of the political sphere were legally permitted, or thought to have been legitimate, under the State of emergency. In India, Sri Lanka, Pakistan, Malaysia, Indonesia and many Asian Societies -- not to mention the African, Latin American and Middle Eastern examples-- such blatantly arbitrary practices of the State were legitimized by regimes precisely on communitarian and developmentalist arguments. "Freedom of the wild ass or food for the hungry was the rhetorical choice offered to citizens by regimes whose developmentalist approach to governance presupposed political obedience without dissent or reS1Stance.
32

Universality Vs. Cultural Relativism in Human Rights The Debate and a Response
In a way, socialist and Left-nationalist discourses of collective rights failed, at this particular conjuncture of State formation, to present a critique of the state. Hence their capitulation before, or the collaboration with, the developmentalist state, the horror of which was effectively camouflaged by socialist or economic- nationalist rhetoric. Thus, the individual-based rights discourse, in its political genealogy had firm domestic -- or indigenous, as some would like to say -- roots in the struggle for democracy and political justice at a historical moment when the space for that struggle had to be carved out anew. The cultural relativist argument, poor in political theory, fails to acknowledge this general facet of State and civil society formation in the non-liberal democratic world. To reiterate, it is the individual rights approach which introduced to the democratic political discourse in the non-Western world such key modcrnist concepts as accountability of the state to society, qualified social obedience to the state conditioned on the shared sovereignty between the state and the citizen and civil Society as a sphere of political autonomy. This it could achieve precisely because of its emphasis on civil and political rights.
Conclusion
The cultural relativist approach to rights, the attraction of which is located primarily in the simplicity of its argument, provides a passionate response to the universalist argument in a context of globalization of capitalism. Being a passion without political theory, it generates nationalistic emotions and forces into the defensive the universalist discourse of rights which has historically originated in the West, propagated by the Western liberals, seeks to de-legitimize the political rights-based discourse, aligns itself with the contemporary developmental state. The dilemma of the universalist approach to rights, meanwhile, emanates, as cultural relativists repeatedly point out, from the appropriation of the rights and democracy discourse by the contemporary global capital. A political practice of rights needs to be politically autonomous from both the state and the global capital. To some extent, the cultural relativists have a point; the universalists have not theoretically disengaged themselves from the global capital and the globalizing state. Establishing this autonomy is the immediate theoretical challenge facing the universalist discourse of human rights.
33

Page 22
.2 2الأمر
Bibliography
Cerna, Christina M. 1994. “Universality of Human Rights and Cultural Diversity: Implementation of Human Rights in Different Socio-Cultural Contexts, Human Rights Quarterly, Vol. 16, No.4 pp.740-752.
Galtung, Johan, 1992, " The Universality of Human Rights Revisited: Some Less Applaudable Consequences of the Human Rights Tradition,” in Asbjorn Eide and Bernt Hagtvet (eds.), 1992, Human Rights in Perspective: A Global Assessment, Oxford: Blackwell Publishers.
Howard, Rhoda E. 1990. “Group Vs. Individual Identity in the African Debate on Human Rights,' in Abdullahi Ahmed An-Naim and Francism. Deng (Ed), Human Rights in Africa, Cross Cultural Perspectives, Washington, D.C.: The Brookings Institution.
Muzaffar, Chandra, 1993, Human Rights and the New World Order
Penang: Just World Trust.
. . . . . . 1995, "From Human Rights to Human Dignity,” in Bulletin of Con
cerned Asian Scholars, Vol.27, No.5, pp.6-8.
Van Ness, Peter, 1995, "Debating Human Rights: The United States and Asia,” in Bulletin of Concerned Asian Scholars, Vol.27, No.5, pp.3-4.
34

Deconstruction is Revolution if Revolution is Body Aching
Yolanda Foster *
Revolution
Since the images you demand
cling to me I cannot form my own image.
I am forced to live by your images, I am always living like that,
SO I understand revolution is really body aching
Nishi Junko
ender is an incessant project, a daily act of reconstruction and interpre ation. As a locus of cultural interpretation the body is a material locality that has been located and defined within a social context. Part of the project of deconstruction, according to Spivak, involves the overturning
Ms. Foster is a doctoral student at the school of Oriental African Studies, England and was a
Resident Researcher at the Social Scientists Association, Colombo.
35

Page 23
'മd لار.
category and organizing concept, and women's experiences as occupiers of systematically inferior positions in all spheres of life provided the historical identity upon which feminist epistemology and politics have been based'. Now the differences among women are asserted in the wake of a homogenizing Eurocentric sisterhood. The trend is to dismantle a feminism whose inclusiveness is perceived as enmeshed in Western universalism. Yet feminist theory remains a useful tool to challenge the basis of gender asymmetry. The question to ask is what kind of theory is useful and why. This paper argues that feminist theories of discourse are key to the feminist project of changing social relations.
It is important to recognise that a key aspect of women's oppression is how femininity becomes culturally produced as a position of marginality. The female body "becomes" female through discursive practice which inscribes gender hierarchies on to the body. The construction of "woman" and the role of discourse plays a role in how women are positioned and become positioned in the Social matrix.
In common-sense language, a discourse is simply a "coherent or rational body of speech or writing; a speech or sermon". But here the term is being used in a specialised way. A discourse is a group of Statements which provide a language for talking about -i.e. a way of representing a particular kind of knowledge about a topic. When statements about a particular topic are made within a particular discourse, the discourse makes it particular to construct the topic in a certain way. It also limits the way in which the topic is constructed.
A discourse does not consist of one statement but of several statements working together to form what the French theorist Michel Foucault (19261984) calls a "discursive formation'. The statements fit together because any one statement implies a relation to all the others. "They refer to the same object, share the same style and support" a strategy... a common institutional... or political drift or pattern" (Cousins and Hussain, 1984:pp.84-5). Discourse is about the production of knowledge through language. But it is itself produced by a practice, discursive practice" - the practice of producing meaning. Since all social practices entail meaning, all practices have a discursive aspect.
| Various writers have critiqued this including; Jane Flar, Anne Marie Goetz, Sandra Harding, Julia Kristeva, Audre Lorde, Gayatri Chakravority Spivak, and Trinh T. Minh-ha.
I have quoted at length here from an explanation by Stuart Hall, useful for its clarity. Hall, Stuart (1994) "The West and the Rest: Discourse and Power"p291 in Formations of Modernity, eds Hall, S. & Gieben, B., 1994, Polity Press, Cambridge.
36

Deconstruction is Revolution if Revolution is Body Aching
To chart the emergence of different discourses and the change in them involves a process of deconstruction. The process highlights points of location and the inscription of subject-positions for women which impact on their behaviour. This is why feminist theory which is committed to exposing the nature of discursive practice is useful. A theory of discourse helps us understand how people's social identities are fashioned and altered over time. An understanding of discursive practice helps us to recognise that the social matrix in which women are socialised and in which expectations are forged is not only about allocative (material) resources but also about authoritative (powers, Sanctions, privileges) resources.
Analysing how a discourse is created can shed light on how, under conditions of social inequality, Social groups in the Sense of collective agents are formcd and unformed (Fraser, 1992:51). Discourse operates on different levels-at the institutional level, through social practices and on the self. Feminists should be interested in how a discourse defines and produces subjects and objects. A focus on discourse rather than ideology allows us to recognise the multi-layered nature of power. Commonly understood norms and Sanctions of society intrinsically involve relations of power. However it is difficult to isolate that power since the discourse through which power operates is multifarious. Certain norms sanctioned by discourse give us the procedural skills for managing social life. We find liberation in discourse as well as discipline. After all discourse helps us to structure the world. How did you learn to speak in the first person singular?
I should like to note that although I use the singular word "discourse" I understand "discourse" in the context of a relation within a Set of discourses. The plural becomes singular through power struggles-contestations over language and practice. A discourse becomes dominant, foreclosing the Voices of other discourses. The dominant discourse is the hegemonic discourse which at this moment in history remains a patriarchal discourse. The hegemonic, patriarchal discourse produces discrete binary opposites for "male" and "female". How are those binary terms "male" and "female" constructed as oppositions. What kinds of cultural roles become ascribed to females, how do these roles change and why are these roles accepted or rejected?
The study of signification helps us to understand these questions. One
concrete mechanism of the function of power is the social construction of subjectivity (signification).3 A deconstruction of sexually constructed gender iden
37

Page 24
,á الأر
tities produces an understanding of the mechanisms of sexist oppression. Feminist philosophy has shown that the universal subject is a myth. The subject is multiple, part of a process which is shaped by various variables of definition, gender being a major axis in Subject-making. How then does gender signification structure social relations? What is clear is that all agents are located in a power network. Subjectivity is therefore positional as opposed to fixed. Although power has a structural base, the relationship between agents and their Social practices to the constitution of Society is reflexive. System contraints affect gender identity but agents also have capability. Power is not deterministic, is not "out there" malevolently wielding its power over people. Power is a multi-layered network which is in a reciprocal relation to subjectivity. The acquisition of Subjectivity is both material (institutional) and discursive (symbolic). What is clear is that gender is an overdetermined source of individual subjectivity. This overdetermined source of individual subjectivity can be read as a Subject-position.
Feminists' aim in investigating the production of subject positions does not mean that this is the locus of subordination. The "production" however can open our eyes to the way in which subject positions impose and petrify traditional gender divisons. It is impossible to proffer a simulacrum of the power network. What we can do is work towards transforming the power principle. Social relations do not inevitably allocate women a subordinate status. As knowledgeable agents women can shift the power principle of the system. Instead of a tendency to power over women's agency and power over other structuring aspects of identity after all are the way people struggle for political rights. What we need are radical ways of reconstituting subjectivity. This is a move away from binary thinking dictating "knowledge" of sexual differences. Peo
Signification refers to the way in which subjectivity is constituted. We are simultaneously social and natural. Biological consciousness is one aspect in the recognition of an "I" with a gender identity. Language plays a key role in shaping the symbolic (discursive) world. Signs come to stand for something. "Woman" being the sign for female gender identity. Signs are made up of signifiers (the image or sound) and the signified (the meaning). The relationship between the two being arbitary. Signs affect the formation of subjectivity, since they pinpoint relational structures in how we make sense of social relations in an unconscious way. The human infant enters into human time by means of a process of translation, in response to the demand of the signifier to be translated. In the allocation of sexual agency the process of translating the signifier of 'woman" or 'man' will have profound implications for gender identity. In order to mediate one's relations with others in the social world it is necessary to take signification seriously. Think of how upset parents are if their female progeny resolutely dresses like a male, has short hair and plays "rough." games. Or imagine a heterosexual date situation in which the female partner asserts "I'm not a 'woman'. Language, categorisation and signs are some of the structuring relations affecting social relations.
38

Deconstruction is Revolution if Revolution is Body Aching
ples' experience is mainly intersubjective. Women like men are embedded in social life and need to negotiate with social practice. The relations between men and women are important. Social and Sexual agency could become allocated on a more mutual basis forming the ontological grounds for political equality. Understanding signification is critical for developing a praxis based phenomenology of social life. Increased mutuality in relations and the allocation of power could transform the patriarchal status quo.
At present there exists a power principle inherent in the way social and sexual agency are allocated. "Woman" is a social and political category which alters at different historical junctures. Up until now it has been predominantly produced as a position of marginality. This "woman" is merely the signifier for a range of different women who may act in different ways to their signification. The crucial point to grasp however is that signification affects all women since it is through signification that women become constituted as a coherent group whose identity is defined in relation to men. The gender identity of men and women as masculine and feminine subjects is socially consituted in and through certain sites, behaviours and practices. Over time social conditions and political status of women have changed. With these changes have come shifts in femininity. If femininity is subject to change then femininity is not a riddle or an essence. Femininity is a Social construct. As in Simone de Beauvoir's classic invocation "one is not born but rather becomes a woman".
Gender is an incessant project, a daily act of reconstruction and interpretation. As a locus of cultural interpretation the body is a material locality that has been located and defined within a social context. Various social practices act on gender to make it, in Monique Wittig's words, a norm that women Struggle to embody. "Woman" thus becomes a false universal flattening out the diversity of women's experiences into an insipid, singular essential. The result being oppressive cultural practices resulting in social and economic inequities.
If we explore the material, social and psychic pressures which feed into the constitution of women's subjectivity we will be able to identify concrete mechanisms of the way power functions or trajectories of oppression. The construction of sexual difference being of key significance, how masculinity and femininity are constructed plays a key role in the expectations and opportunities available to women. How do women negotiate their own subjectivity when they are enmeshed in ideological construction?
39

Page 25
However, if "woman" is culturally and historically specific, then "woman" is amenable to deconstruction and reconstruction. To feminists this knowledge allows for new politics of sess-creation. If women themselves start to speak up and define themselves they can become more active in positioning themselves in the power network.
Any theoretical position has its weaknesses and cannot do justice to the range of women's expression, their different life experiences and choices. What I am interested in however is inviting readers to explore feminist theories of discourse to open up a dialogue about the power dimensions of domination and marginalization that underScore the concrete class/caste and race positions of women.The aim is to focus on the process of signification-on how women become constituted as a coherent group whose identity is defined in relation to
C.
Part of the project of deconstruction, according to Spivak involves the overturning of implicit hierarchies established in the binary to "discover the violence" (Spivak, 1990:8). The question is do we just gape at the violence or do we try to provide a position for critical intervention? Whilst feminist political practice requires a position from which to operate this cannot be one-dimensional. We need to be sensitive to a homogenizing Western feminism which eludes difference. Unlike some anti-Enlightenment figures however I feel that we can challenge the category of "woman" whilst retaining some grounds for solidarity. We must strive for essential gains despite our critique of essentialism. Feminists cannot afford to become ghettoized. In place of ethical abdication we must improvise wisely. We must claim a space-in-between the hegemonic discourse without romanticisation or guilt. How we "live as ideas" (or in academic jargon how we are discursively constructed and construct ourselvcs) is an area which may help us develop Strategies for social change. Here's to revolution as body aching or feminism as the shedding of skins which do not fit
40

Deconstruction is Revolution if Revolution is Body Aching
Bibliography
Armstrong, Isabel (ed), 1992, New Feminist Discourses, Routledge, London.
Barrett, Michelle 1980, Women's Oppression Today: Problems in Marxist Femi
nist Analysis, Verso Press, London.
Bradiotti, R 1994 Nomadic Subjects, Columbia University Press, New York.
Butler, Judith 1990 Gender Trouble/Feminism and the Subversion of Identity,
Routledge, London.
Connell, R.W. 1987 Gender and Power, Polity Press, Cambridge.
Simone de Beauvoir (1984, 1949) The Second Sex, translation,
Hammondsworth, Penguin.
Elam, Dianc 1994 Feminism and Deconstruction, Routledge, London.
Foucault, Michel, 1980 b History of Sexuality, Volume 1, An Introduction trans.
Robert Hurley (New York:Vintage Books)
41

Page 26
മ الألم.
Fuss, Diana 1989, Essentially Speaking. Feminism, Nature and Difference,
Routledge, New York.
Harding, S 1991 Whose Science? Whose Knowledge?, Open University Press,
Buckingham.
Hooks, Bel 1981 Ain't I a Wonnan. Black Women and Feminisin, South End
Press, Boston.
Millet, K. 1969 Sexual Politics, Garden City, NY: Doubleday.
Lorraine Radtke H. & Hendrikus J. Stam 1994 Power/Gender: Social Relations
in Theory and Practice, Sage, London.
Nicholson, Linda 1990 Feminism/Postmodernism, Routledge, London.
Spivak, Gayatri 1987 In Other Words, Routledge, New York. 1990 The Post
Colonial critic. Interviews, Strategies, Dialogue, London, Routledge.
Trinh T. Minh-ha 1989 Native, Other, Indiana University Press, Bloomington.
Wittig, Monique 1992 The Straight Mind and Other Essays, Harvester
Wheatsheaf, New York.
Weedon, Chris 1987 Poststructuralist Theory and Feminist Practice, Blackwell,
London.
42

Still Struggling.....
Shiranee Mills
I want to look up..... Yet structures red hot crumble on me, Pushing me back, Smothering me. I strain under the debris, still struggling; Again they laugh and laughter echoes, Through the hollow walls of time.
Pandora screeches her ugly laughter, And the clawing, toady things slowly creepout Surging, stumbling - heralds of evil. Kooni chuckles away, cunning, crafty, cruel, Telling vicious tales, poisoning, tainting, Ushering cnvy, cankered relationships; The Witches Three, brewing their bloody concoctions, Cackle, cackle, cackle, screech and cackle, Dump fair Macbeth into the cauldron.
* Shiranee Mills is attached to the Women's Education and Research Centre as a Senior Project Officer.
43

Page 27
,á الأر
I want to look up..... Yet myths of time bear me down, Prowling, seeping, trickling into present reality, Taking gruesome forms, Crushing me, scarring me. Eve still plucks a thousand apples In a thousand Edens.
"Come out' they shout, "why are you whining? Twentieth century wants you out of those depths.' They shout, words falling down, down, down, To reach me,
“Come out, out, out, out...”
Don't you see
I cannot - till you stop measuring me by the legends you quote. The legends are yours the myths are yours You put me here.
So bend down, reach out, it's you who must pick The rubble off me, Brick by brick, stone by stone; Laid, reinforced, concrete. The weight of eons so heavy upon me Must now fall away.
I want to breathe, to live, to look upCome unbury me.
44

A Chinese Woman Breaks the Silence
his letter was written by a Chinese woman, who prefers to remain anony mous. It was distributed at the NGO Forum in Beijing.
As a Chinese woman, I am extremely happy to attend the NGO Forum on Women in Huairou, and I feel empowered by women from all over the world. However, I feel sad for my sisters who are excluded from the Forum. To say that the All Women's Federation of China is an NGO is a joke: everybody knows that the All Women's Federation is funded and controlled by the Chinese government. Many other Chinese NGOs don’t meet the criteria for non-governmental organisations. Unfortunately, I think the majority of Chinese NGO delegates are from central and local branches of the All Women's Federation and other government related or controlled organisations. In fact, real non-governmental groups of Chinese women (groups of women who do not work directly for the government, such as women peasants and workers) are excluded from the Forum.
Courtesy - Women's Global Network for Reproductive Rights, Newspaper 53-January - March 1996, From : Wanawake, the Netherlands, November 1995
45

Page 28
M2 മ
At the NGO meeting, Chinese delegates give the impression that Chinese women have already obtained or already have been granted equality, and that they don't have the problems that women from other countries are facing, Such as violence against Women, inequality in employment, and sexual harassment. It appears that most of the Chinese NGO delegates see it as their mission to disseminate this image of China to the world. As a result, the issues and problems of Chinese women are not exposed here, nor discussed. This means that Chinese women, whether from the government or not, are silenced by the male dominated government.
As a Chinese woman, I know we are facing increasing inequality caused by this male dominated society. Some old problems are not yet solved, and new ones have arisen already. It is true that the women's liberation imposed by the communities party improved women's living condition and social status. However, the liberation didn't eliminate the institutional discrimination of women that prevent them from getting access to the decision making class. The liberation didn't give women the freedom to develop their own feminist theory independent of the communist one. It resulted in a triple workload for women; house work, reproduction and work outside the house. To gain a limited liberation, women have to work much harder than men and their domestic and reproductive responsibilities are still used as excuses to prevent them from getting promoted or obtaining a job that they are qualified for. Since the economic development, many old problems that were temporary controlled by the communist government reappear. Pornography, prostitution of women, and Sexual transmitted diseases are increasing in an alarming speed. Modern concubine is secretly emerging. Many middle aged women are laid off as victims of the restriction of the labour force. An increasing amount of women coming from remote regions in China are sold for a few US dollars to be some poor men's wives and be poisoned, raped and tortured by a husband who believes that she belongs to him like a thing he bought.
Defining some government branches as non-governmental organisations is a way to exclude the real non-governmental groups. Denying the accessibility of non-governmental groups helps to cover the issues and problems of Chinese women. These are violations of the rights of women to attend the NGO Forum and to freely express themselves. Covering women's issues and problems can only facilitate the existing inequality. This is the reality of Chinese
46

A Chinese women breaks the silence
Women. They are tightly controlled and suppressed by the male dominated government. If an officer from the All Women's Association of China does not have the freedom to discuss women's issues, how can a common woman feel free to report a rape or complaint about sexual harassment by her male boss (the system makes women feel ashamed for the crime of men).
This Fourth World Conference on Women calls for women's action for equality, development and empowerment. But how can Chinese women take action if their issues are not even raised? How can Chinese women take action under such tight control?
I know that many women are suffering from existing inequality. Why can't they come to this conference to tell their stories to raise awareness about their issues and problems for the purpose of preventing similar tragedy to other women? And although I know what is going on, I don't feel free to express my criticism, not even in this place.
47

Page 29
á 4%2ره
Gender Asymmetry in Labour
Shiranee Mills
labour. It takes the reader through the build-up of society's practices
which culminated in the establishment of patriarchy. The paper thereafter explores patriarchy's infiltration into the labour world. It also projects dualisms in society's understanding of the word “labour'.
T paper seeks to give a theoretical focus to gender asymmetry in
To understand the term “gender asymmetry', one would do well to study the following piece of writing by a 15th century Italian author:
“Men are by nature of a more elevated mind than women. They are more Suited to Struggle with arms and with cunning against the misfortunes which afflict country religion and one's own children..... Women on the other hand, are almost timid by nature soft, slow and therefore more useful when they sit still and watch over things. It is as though nature provided for our well-being arranging for men to bring things home and for women to guard them. (Alberti, 1969, 207)
48

Gender Asymmetry in Labour
The dichotomous ideology of asymmetry in gender had its roots definitely in such conclusions- the private domestic world of women contrasted with the public world of action, where men order and control nature.
To these traditional assumptions Darwin gave theoretical and Scientific basis with his belief that men were "more courageous, pugnacious and energetic than women and had a more inventive genius.”
Such thoughts and concepts have developed through the ages and variations of the same dichotomy have been and are evident in different levels in different societies. The assumption that there are differences between the sexes, an assumption which assigns different tasks to women and men, is found in almost all Societies. It is said that the Segregation and confinement of women in Mid-Eastern societies reveal the belief that the domestic sphere is the place for women. On the contrary in Africa, the Yoruba women are the traders in market places and are the linchpins of the economy. (Rosaldo, & Lamphen, 1974: 20)
Whatever the practices and beliefs in culture and society, women all over the world, are always placed in a subordinate position. There is a universality in woman's subordination. Sherry Ortner analyses this question in her essay - "Is Female to Male as Nature is to Culture' - and gives us the view that woman - for her to be considered of lower value - must be associated with something which every culture devalues. Ortner's view is this “there is only one thing that would fit this description and that is "nature', in the most generalised sense. (Ortner, 1974, p.72). She explains here that all cultures recognise and form a distinction between human Society and the natural world. Culture, she says, attempts to control and manipulate nature, thus transcending nature, dominating nature, in order to use it for its own purposes. Culture is therefore seen as Superior to nature and also extractive of nature. Thus Ortner Suggests that women are Symbolically associated with nature, while men are associated with culture. It is this belief which created the notion that woman could be controlled and dominated by man.
Woman is thus closely linked to nature because of her reproductive functions of giving birth, rearing, nurturing (all acts close to nature). She is limited to certain social functions because of pregnancy, birth, lactation and nurturance - all roles which are associated with women. These functions which she performs limit her and confine her to her domestic domain. Man thus
49

Page 30
M2 άπει
becomes associated with public and social life while a woman is seen as limited to domestic life. Ortner in her essay emphasises that women are not any closer to nature than men; she says that they only "appear closer to nature'.
Although sexual ideologies, bcliefs and stereotypes vary greatly in different societies, certain symbolic associations between gender are common to these societies. Men are associated with up, right, high, culture and strength while women are associated with their opposites. These are cultural constructs which place woman on a lower place. We could assume therefore that it is the ideological link between woman and her reproductive functions which limits her in her social role.
Engel, talks of a time when women were Supreme in the familial context. The supremacy of women during this early period of barbarism was thought to be a result of the fact that fatherhood was impossible to determine with any certainty because of group marriages. Descent, in such societies, could be defined only through the mother. Therefore Engels says "There was a highesteem for the women, that is for the mothers' (Engels, 1972, pp. 60-61)
With the introduction of monogamy, the transition from matrilineal to patrilineal descent emerged. This was closely linked to the growth of wealth and property. Engels thus concludes that with the growth of wealth and property, new social relationships surfaced.
"According to the division of labour then prevailing in the family, the procuring of food and the implements necessary thereto and therefore, also the ownership of the latter, fell to the man; he took them with him in case of separation just as the woman retained the household goods." When the man's task expanded with domestication of animals and the advancement of better agricultural implements and techniques, his wealth increased and thus gave him " a more important status in the family than the woman.' (Engels, 1972 :66)
Thus the increased wealth and status of the man enabled him to "overthrow the traditional order of inheritance in favour of his own children.' With this it is said, patriarchy was established and this has been defined as "the world - historic defeat of the female Sex' (Engels, 1972: 67-68 (emphasis in the orginal) Engels thereafter looks at the man of the family as “the bourgeois; and the wife as the proletariat.” (p.82) - for here the man with the expansion of
50

Gender Asymmetry in Labour
property and the accepted patrilineal System gains dominance over woman. The original role of man the hunter, provider and woman the child bearer, food producer, weaver etc., transcended into different roles with different tones, with the accumulation of private property. If woman was discriminated against in earlier times, with man's accumulation of wealth and property, (owing to patriarchy), her subordinate position became more intensified or magnified. The term patriarchy' has been carried into this era and looms large. Patriarchy it could be said refers to the systematic nature of men's power.” (McDowell and Pringle, 1972: ()). In Hartmann's words patriarchy is :
A set of Social relations between men, which has a material base, and which though hierarchical, establishes or creates interdependence and solidarity among men that enable them to dominate women. (Hartmann, 1981: 14-15)
With the emergence of patriarchy and industrial capitalist production, where man obtained the livelihood through accumulation of wealth, woman's housework lost its significance. This brought about the separation of home and work spatially. Woman, who because of her reproductive functions and child - rearing was assigned the household tasks of Washing, ironing, running Small shops, small scale food production and other ways of earning a small income, became insignificant; her household tasks and her contribution to the cash economy became invisible in the face of formal production in the factories.
Thus the sexual division of labour - ie the allocation of tasks on the basis of sex - pushed woman further and further into the background.
"The sexual division of labour cannot be understood in purely economic terms. It has sexual and Symbolic dimensions as well. It is not imposed on people but comes as part of a Social package in which it is presented as right, natural and desirable. Our identities as masculine or feminine beings are bound up with it.” (McDowell and Pringle, 1972:124)
Although woman's work in the home goes financially unrewarded, the tasks that she performs at home could be categorised under work/labour. Feminists of the 1970s and the 80s criticised the narrowly based definitions of work which took into consideration only employment and productivity in the market as work. They argued that equal to the production of goods were the tasks of reproduction - reproduction of children, reproduction of existing social relations. They concluded that these too should come under the term “Work”.
51

Page 31
á لار
Domestic labour is still the woman's sphere, as it were, although technology and modern conveniences have made the work lighter in many developed and developing countries. (This is not true of all Societies - for fuel gathering and water collection from distant places still go on in many parts of the world.) Through all these technological changes and modernisation, the one thing that has not changed, it could be said is the fact that it is still accepted that the domestic chores should be done by the woman.
Ann Oakley in her article "What is a Housewife?' recognises four characteristic features of the housewife-role in modern industrialized society:
1) its exclusive allocation to women, rather than to adults of both sexes; 2) its association with economic dependence;
3) its Status as non-work as opposed to economically productive work 4) its primacy to women; - its priority over other roles performed by her.
(Oakley, 1974)
These four characteristics are important to our understanding of gender asymmetry in labour and could be said to be the root causes for gender asymmetry in labour. When household chores are allocated solely to woman, she is burdened not only with the manual tasks that she performs but also with the heavy responsibilities that go with it, as child nurturing, their welfare etc. In the case of the working woman the household chores become a double burden to her, for she goes out to work during the day and comes back home to resume her duties as housewife.
The second feature of economic dependence of the woman places her in a subordinate position within the family.
The third factor which defines woman's household tasks as "non work' not only places her once again in a subordinate position but "devalues' the work that she does at home.
The fourth feature recognised by Ann Oakley shows woman as placing her family and the care of the husband and children as her primary role and concern. This belief not only limits the woman to the household but prevents her from choosing a career of her own- for she is expected to choose work which would give her time to care for the family. In such a society where the
% Ꮌ Ꮸ 8.
woman is expected to excel in the roles of "housewife', "mother' and "wife' -
52

Gender Asymmetry in Labour
the woman has little time or desire to pursue education or the career which she desires.
Although the world spatially separates the "home' and "work', when women enter wage labour, the concepts of patriarchy and the Sexual division of labour are carried into the labour market, thus once again causing gender asymmetry in labour. Roisin McDonough and Rachel. Harrison in their work titled “Patriarchy and Relations of Production', have worked out that patriarchy not only dominates the process of production but also permeates class relations. (Kuhn & Wolpe, 1978: il-41) They point out that patriarchy could be defined as a factor which controls woman's labour (both in the household and in wage labour) and that this control is closely and crucially connected with control over her sexuality and fertility.
The increasing participation of women in wage labour and the studies conducted over the years present three major concerns which exemplify gender asymmetry in labour:
) Besides the fact that woman is excluded from the paid work force owing to the fact that she is doing it in the domestic sphere, woman while in the labour work force has been treated as "invisible, as invaders, whose
proper' and primary place is at home.
2) Men and women still work in different types of jobs, with women "clustered round in a narrow range of occupations’ - leading to occupational Segregation and wage differentials. (McDowell and Pringle, 1992: 154)
3) Chhaya Datar in her study of a number of females in the work force recognises besides other factors, the sexualisation of skills, where the work categorised as unskilled and therefore is underpaid, is often based on very intricate skills acquired in housework. (Chhaya Datar, 1982)
Further, Janet Siltanen in her article titled, "A commentary on Theories of Female Wage Labour' presents a statistical report, which explains gender asymmetry existing in labour. Firstly she observes that the proportion of the labour force that is female increased from 30 percent in 1911 to 40 percent in 1974. She recognised two marked features in this fact - that there has been an
53

Page 32
increase in the number of married women entering the labour market and that a greater proportion of women workers work part time. The second feature that she observes is that although there has been an increase in the number of women in wage labour, it has had little impact on the segregated nature of women's labour. She further states that according to EOC News Reports while women's gross hourly earnings rose from 63 percent of men's to 75 percent from 1970 to 1976, they rose only 4 percent in 1977 and have dropped back to 74 percent in 1978. These factual statements gleaned from studies conducted by different researchers, prove the existence of sexual division of labour and inequality in wage labour - in other words gender asymmetry in labour.
Labour analysts and feminist theoreticians have developed several theoretical models which analyse the causes and the impact of gender asymmetry in labour. One theoretical model used for this analysis is termed the Human Capital Theory.
Human Capital Theory:
The Human Capital Theory explains wage differences by reference to productivity. The term Human Capital could be described as education, training courses, job experience and job training. The possession of these factors enables a person to enhance her/his productive capacity. Individuals thus make investment in their potential productive capacity through acquiring education, training and experiences. Once they increase these, they increase their productivity in the labour market and thereby increase their wages. When a woman enters the labour market her productivity is low because she hasn't had the time nor the inclination to increase her human capital, such as further education, training etc., owing to the fact that she has been preoccupied with child bearing tasks at home. The time she has spent on child bearing and rearing, decreases her productivity in the labour market. Consequently her wage decreases.
Human Capital Theorists argue that the woman's primary choice in her child rearing role and household duties which it entails, results in her human capital being lower than man's, thereby her wages. There are two ways in which this is possible argues Janet Siltanen :
54

Gender Asymmetry in Labour
1) Woman on the assumption that she would be spending more of her time
with the family wouldn't invest in human capital.
2) Woman, when she takes time off from paid work to tend the familyallows her already acquired human capital to deteriorate. Thus her skills become rusty or obsolete. When she returns to work she is placed in a less competitive position. Does this theory sufficiently explain sexual divisions in labour and the inequality in wages? Janet Siltanen concludes thus :
'even though human capital theory takes all these into account, there still remains a substantial wage difference between men and women that is unexplained by productivity variables'.
( Siltanen, 198l:27)
She quotes studies which have discovered that many who get low wages and are unemployed have a considerable amount of human capital. She also says that women of a certain job category are "over qualified' for that particular job and are often more qualified than the men who occupy a similar job.
It is sometimes argued that women enter the wage labour market, in order to get pin money and that they have the luxury of opting between working for a wage and working for the husband and children. The argument fails to consider that there are women who are the sole breadwinner' of the family. Besides, a double wage (by the husband and the wife) has become a necessity of modern day. Here the argument that women's wages are pinmoney has no justification. We could also argue that the human capital theory is “sexist' because it only counts as production those skills which are rewarded financially, thus many skills women possess go unrewarded and unidentified.
Labour Market Dualism
Labour economists have developed another theory of discrimination which either Supplements or replaces, the human capital theory. The dual labour market as its name reveals distinguishes two labour markets, a primary and secondary sector. Janet Siltanen describes this theory through an explanation of the internal market. An internal labour market she says operates within an administrative
55

Page 33
'Ze هلال
unit. It involves the governing of wages and job allocation by a fixed system of administrative rules and procedures. Certain jobs in the external market act as ports of entry into the internal market. The primary Sector has an internal labour market structure and the secondary sector lacks this or has a very undeveloped internal labour market structure.
The primary market is characterised by the following features:
а) entry is restricted to few lower level jobs.
b) promotion ladders are long
c) high wages are paid to ensure worker stability
d) opportunities for advancement are given
e) good working conditions are provided
f) job security is evident in the primary market
g) on the job training is offered, mostly at company costs
On the other hand, the secondary market has the following factors: numerous entry ports into the secondary sector; promotion ladders are long and sometimes nonexistent; Low wages are given in order to discourage worker stability; there's little opportunity for advancement; Dual labour market theory thus attributes woman's limitation to the secondary sector because of the priority she gives to the home: as a result she has "low commitment to work, low promotion initiative and intermittent employment'. On the contrary the dual labour market theory also suggests that the woman may leave paid employment, not because of a "Moral choice between family and work but the reality of her position as wage labourer. (Siltanen, 1981:31)
The dual labour market theory is inadequate in its, explanations owing to the fact that job structures are viewed in fairly static terms' (This conflicts with the de-skilling which occurs in the labour market). This theory also does not address the wage differentials between men and women within the secondary labour market, nor does it consider the wage differences between male and female, within the primary market.
Radical economists have given a more dynamic account of this model explaining it as Segmented Labour market theories. They argue that these different labour markets arise owing to the employer - intention of keeping the labour market segmented and divided in order to counter working-class militancy. When the workforce is divided into different categories their
56

Gender Asymmetry in Labour
experiences too vary thus unity too cannot be achieved. Hence, labour markets are Segmented by class, sex, age, race and ethnic origin.
Another theory which underlies gender asymmetry in labour is the theory of de-skilling and the Reserve Army of Labour. This theory was first introduced through the publication of Harry Braverman's book titled "Labour and Monopoly Capital the modern technology was degrading and devaluing the dignity of work - by taking away traditional, old craft-skills and drawing more and more workers into an enlarged proletariat This was of particular interest to those involved in feminist studies because the book paid attention to the proletarization of clerical work. (McDowell & Pringle, 1972: 155) Janet Siltanen too explores this book, relating it to feminist studies. Braverman states that clerical jobs were at onc time highly skilled and highly paid (Clerical work was predominantly done by women) but with machinization such jobs have been deskilled. These jobs have now lost their high value owing to modernisation and therefore receive low wages.
"Compared to its previous position of relatively high pay, job autonomy and skill, clerical work today is highly mechanised, highly supervised, highly routinsed and de-skilled' (Siltanen, 1981:34)
Feminists taking Braverman's theory added a gender dimension to it and Said that labour processes are shaped by struggles between men and women. (They derived this from Braverman's theory that changes in the organisation should not be treated simply as technological innovations based on capital's search for higher profits, but as the outcome of struggles for control between capitalists and workers). The people who are discarded because of mechanisation, move to other jobs which are labour - intensive, jobs which are not mechanised, or they join the ranks of the unemployed. Thus the excess labour from industry and female labour from homes become the occupants of the reserve army of labour as workers who get Sucked into the labour force during periods of expansion or crises, and spewed out again when circumstances change.” (Moore, 1988: 13). The fact that women form a part of the reserve army of labour is linked to the idea that women receive lower wages than men. The argument attached to the theory of the reserve army of labour is that married women are dependent on their husbands and that they could be laid off work in times of crises and that employers could lay them off comfortably assuming that they would be looked after by their husbands. Thus these employers exploit the assumption that the waged work of married female employees is Secondary to
57

Page 34
their roles as wives and mothers; this results in women being paid wages so low that they do not even cover the costs of reproducing them as workers (Beechey, 1978: 155-97) Janet Siltanen uses the theory of de-skilling to explore another aspect of female labour. She states that Domestic labour is being constantly capitalised. She says that domestic labour through becoming machine oriented, "sets frce' female labour in the home for its use in capitalist production. (Siltanen:36)
Thus the concept of the reserve army becomes useful in explaining certain aspects of female labour. The theory it could be said fails to consider the fact that the reserve army, especially women as a reserve army are not a homogenous lot, they are differentiated in that Some are dependent on incomes other than their own, their reproductive life cycle and the kind of work they form a reserve for.
In conclusion one could say that just one theory is inadequate to explain all aspects of female wage labour. Female wage labour is not static - its conditions and features are bound to change with time; with different social conditions it attains different tones, as it were.
"The comparative pictures that emerge from studies of women's waged employment around the world is one of variation and complexity; there is no single explanation which can be said to characterise the relationship between women's work in the labour force and the sexual division of labour in the home." (Moore: 113)
Considering the fact that feminists from a variety of disciplines have consistently and forcefully argued that "family' is the "central site of women's oppression' in society (which of course includes the workplace), it is time that new thinking and concepts were evolved, which would liberate in its fullest sense, the women in the home thereby giving her equality with men not only in the home but also in the workplace. It's only then that the Cairo factory woman's words in Barbara Ibrahim's study of the Women and the Family in the Middle East:
“Work strengthens a woman's position. The woman who works doesn't have to beg her husband for every piaster she needs. She can command respect in her home and can raise her voice in any decision."
v > «», KO « it's then and only then that these words will ring true for every woman.
58

Gender Asymmetry in Labour
References
Alberti L.B., 1969 The Family Renaissance Florence, Columbia, South Carolina:
University of Carolina Press
Beechey Veronica- 1978 Women and Production: a critical analysis of some sociological theories of women's work.'ed. by A Kuhn and A. M. Wolpe - Feminism and Materialism. London Routledge and Kegan Paul
Datar Chhaya- 1982 Redefining Exploitation I.S.R.E. Bombay
Engels - 1972 The Origin of the Family Private Property and the State NY
Pathfinder Press
Hartmann Heidi - 1981 "The unhappy marriage of Marxism and feminism: towards a more progressive union, ed. by Sargent L. Women and Revolution, London: Pluto
Kuhn Annette , Wolpe Anne Marie (eds) - 1978 Feminism and materialism,
Routledge and Kegan Paul
McDowell Linda and Pringle Rosemary (eds) - 1972 Defining Women - Social
Institutions and Gender Divisions, The Open University in association with Polity Press
59

Page 35
'Ze الألم
Moore Henrietta- 1988 Feminism and Anthropology, Polity Press
Oakley Ann - 1974 Housewife; - London Allen and Unwin
Ortner Sherry - 1974 "Is Female to Male as Nature is to Culture?" - I.M.Rosaldo and L.Lamphen (eds) Woman Culture and Society: Stanford: Stanford University Press
Rosaldo M. Zimbalist and Lamphen L. (eds) - 1974 Woman, Culture and
Society. - California: Stanford University Press
Siltanen Janet - 1981 "A Commentary on Theories of Female Labour' Women
in Society (Inter disciplinary essays) The Cambridge Women's Studies Group
60

y Through Women's Eyes: Global Forces Facing Women in the 21st Century By Charlotte Bunch *
The end of this century is a critical juncture for women because it is a time of many transitions. The end of the cold war has brought a hot peace as power blocs shift and economic uncertainty rises throughout the world. As racial, ethnic, religious and gender-based conflicts and violence escalate, questions of women's human rights and women's role in Society have become central. It is both a dangerous time for women with backlash against advances we have made, and a time of opportunity for women to emerge as significant players to determining the future.
The opportunity is there for women to offer new solutions, to build on what they are already doing as leaders at the local level and enter the global policy debates. Women have held families and communities together in times of crisis. They have managed budgets that were inadequate to raise children and have kept people together in times of war and conflict. And yet, as power moves up the ladder from local community to national and international policymaking, women's voices and women themselves disappear. But, in the last few decades we have begun to demand a place at the policy-making tables as well as the kitchen table.
Courtesy Global Center News No.3 Summer 1996
61.

Page 36
M2 Ze
While there is much talk these days about global governance, there is not enough conversation about the undeclared global governance we already have unaccountable by the global economy, where market driven values and trade dictate policies to all nations through the international Monetary Fund, the World Bank, and various military alliances. This has resulted in structural adjustment policies in developing countries, and downsizing and dismantling of social welfare in both formerly socialist countries and much of the industrialised world. Both of these sacrifice human needs and human rights for economic expediency. And everywhere, it is women who suffer the most from these policies because it is women who must make up for the services lost to family and community, as well as continue to work at lower paid jobs.
Both economic and cultural life are becoming more dominated by global market values. Yet, we have no effective global political structures for overseeing these processes. Rather, we are told we either have to accept the “rule' of the global economy with its frequent downsides for women or we have to return to "traditions' which are based in patriarchal culture. The challenge for women is to develop a third way, another option for growth and development that is done within a context of valuing human rights and equality. Some groups are reacting against the global economy and their lack of control over their economic life by clinging to local identities which involve narrow definitions of who they are. A narrow nationalistic ethnic fragmentation grows into separatist enclaves where all 'others are demonized and seen as less than human, as reflected by ethnic cleansing in the former Yugoslavia, the genocides committed in Ruwanda, as well as emerging neo-fascist, racial supremacist forces rising in industrial countries.
The rise of religious fundamentalist movements that take a narrow patriarchal view of religion, whether Muslim, Christian, Hindu, Jewish or others are based on a narrow call for identity that dehumanises "others' who are not members of their religious group. Their sense of commonality is developed in opposition to and seeks domination over others, rather than building a spirit of solidarity, of humanity, and tolerance for those not like oneself. Such reactionary forces, whether nationalistic or religious, or both, all seek to control women to ensure the purity of the race and the means by which it will expand. In that point lies the vulnerability and potential strength of women. Women must refuse narrow definitions and say that there can be diverse cultures and ethnic identities living together, that there can be tolerant religions that don't have to be in opposition to the other, that we can live in solidarity and respect with those
62

Through Women's Eyes: Global Forces Facing Women in the 21st Century
who are different. If women do this, we can be the key to denying narrow fundamentalist movements their source of power and regeneration, and move toward a more humane option.
Those forces that seek to return women to narrow definitions in the family solely as reproducers and caretakers are also agitating against the rights of minorities, whether racial, ethnic or religious groups, gays and lesbians, gypsies and immigrants. All of our humanity is denied when some group gets defined as "the other'. If we accept that any one is less than fully human and therefore deserves to have fewer human rights, we start down the slope of losing human rights altogether. Women should understand this since we live in a male-defined world, where we are still the primary "other' and the definition of issues overlooks our experiences.
Recognising that our experiences have not been included in the political process, women are working toward redefining fundamental questions of our global order - questions of democracy, development, environment, peace, human rights, etc. Looking at these questions from the point of view of women's lives, wc ask how they might be approached differently if they included the concerns of all who have been marginalised by the dominant patriarchal paradigm. Thus we pose alternative social models not based on domination, alienation, and the divisiveness present in the world today.
Central to women's responses to these global forces has been transforming the definitions and interpretations of human rights. The terminology of human rights came initially from the experience of the white propertied European male who did not need to worry about violence in the family or poverty because those were not his problems. Where he felt his humanity most violated was in relation to the state, in terms of rights like freedom of religion and speech. While these issues are also important to women, we are expanding the understanding of human rights to acknowledge that fundamental to all human rights is the right to exist - to food and shelter and freedom from all violence. Women's right to freedom from violence and torture must be defended in the home and in the streets as well as in relation to the state. Many women do not even get to the point where they can exercise their human rights as citizens because they are So oppressed in their homes or by economic inequality that they are unable to even enter the political sphere.
63

Page 37
'(Z هلال
In asserting that women's rights are human rights, we also show how human rights violations are usually gendered - that is, how one experiences the violation is shaped by whether you are male or female, and also by other factors such as class, race and culture. The violations experienced by men are usually considered more important than those most often experienced by women. In challenging human rights to expand to include women, we are also expanding who is seen as fully human or a "citizen'. We need to learn to respect the common humanity and universality of the human rights of every person while also creating space for the incredible multicultural diversity that exists among us so that everyone doesn't have to become like the dominant group in order to have rights. There can be variety in human rights practice but no groups' rights should be sacrificed in the name of culture or diversity. Human rights is a constantly evolving concept that responds to how people see their human needs and dignity over time. Thus, as people exercise their human right to selfdetermination, there will always be a process of both expanding the concept and ensuring that the exercise of rights does not allow for domination over others.
Women's involvement in human rights dialogue is part of the process of breaking away from the polarized options that globalization offers to move back to the past to preserve identity, or to move to the future simply by accepting domination by the global economy. Women must seek democratic structure for global governance, and demand accountability and respect for human rights from bodies, like the United Nations, which reflect the male domination of governments in the world. At the IV World Conference on Women, we sought to put governments and the United Nations on notice that we were watching to see whether they could develop global democratic structures of governance that fully include women or whether women will have to go elsewhere to gain justice and human rights.
For me, the most important global force today is that of women in movement around the world. This global force has many different names - call it feminist, call it womanist, call it women in development, call it women's rights or women's human rights. Each of us has found different terms that describe the reality of women seeking to end patriarchy and bring change. Women are the most important new global force in the world today with the potential to create a more humane future.
64

Through Women's Eyes: Global Forces Facing Women in the 21st Century
For women to be such a force carries great responsibility. We cannot be a movement that thinks and speaks only from our own experiences. Our movement began with the urgent need to put women's experiences on the agenda because these were missing, and women's perspective are still often left out of policy deliberations. But if we don't want to be simply an added-on dimension, we must also bring in others whose voices are not heard - all the diverse women and men whose voices have been muted - so that we show it is possible for this world to hear from all its peoples. There will be conflicts, but we must seek non-violent ways to resolve them that move towards the future and away from the militaristic models of domination in which the world operates today.
The UN world conferences have served as global town meetings where women had opportunities to meet and talk across the lines of culture and nationality. They were also occasions for us to demonstrate our visions of how the world can be transformed by looking at it through women's eyes. We must continue this global dialogue beyond world conferences as we participate more in political life at all levels. We must work for democratic global governance, where policies can be created with a human face that is both male and female, and where all the human diversity of race, sexual orientation, culture, religion, age and physical ability can emerge. Women with this vision must be prepared to help lead the world toward a better future for all in the 21st century.
65

Page 38
/ല്ല
Towards a better model for Women’s pages in Sri Lankan English dailies
Manique Karunaratne*
he metamorphosis in the position of Sri Lankan women in society has
led to the growing recognition of the role of women as catalysts in na
tional development. Speaking of women in media, Gloria D. Feliciano stated that studies have established that in many Asian countries, rural women have functioned as primary sources of information, as well as initiators, legitimizers and decision makers; in introducing change to the village. Available studies have also shown that enlightened, modern urban women do no less as communicators and assume even more prominent roles in modernizing the cities. "The increasing participation of women in the communication media thus assumes great significance for development and should therefore be viewed with jubilation,” she maintained.
* This article first appeared in the Sri Lanka Journal of Communication, a journal of the Department of Mass Communication, University of Kelaniya, Sri Lanka, in 1986. Finding that the contents are still relevant we are republishing this.
66

Towards a better model for Women's pages in Sri Lankan English dailies
A study of the image of women as depicted in the Mass Media channels in Sri Lanka was recently carried out by the Department of Mass Communication, University of Kelaniya. The study pointed to the fact that the image of women was to a great extent distorted, undermined, exaggerated and misplaced.
All the local English dailies in Sri Lanka devote special pages to women, usually on a weekly basis. These “Women's pages' have the potential to play a major role in development communication. However, Ben Bagdikian's observation "Most papers still look as though they are edited on the social assumptions of the 1940s and 1950s - " fits the Women's pages of local English dailies. To make matters worse, even the "Social assumptions of the 1940s and 1950s.' are not local; but totally Western
The contemporary Sri Lankan woman is identified as an individual changing her outlook, life style and image of Self, but the changing woman; in this case the English educated elite may be reading a newspaper that views her as a bucolitally contented simpleton whose most pressing questions are learning the tactics of "Making Your Man feel loved' or finding out the secrets of the "Beauty asset that makes you look younger than your age.'
The Women's pages operate on a stock formula of society, clubs, decorating, furniture, food, cooking, children, husbands, beauty, fashions and Sewing. Whatever ingredients of this formula the newspapers may pick for its female readers cach week, the underlying theme is the attempt to promote Westernization. Many of the feature articles, fashion pictures and recipes are picked from Western Sources. Even the local material is predominantly western in character.
The messages of these Women's pages represent information failure in the context of development. The failure is obvious to the audience and often to the female media planners who produce them. Within that limited coverage such selections present shallow reporting - reflecting fashion in terms of the offerings of the newspaper's top advertisers, without discussing the high cost and poor quality clothing, featuring recipes and eye - catching colour pictures of foreign food, without mentioning the problems of malnutrition and in many instances, reporting the total trivia of embassy parties and fashion shows, as if it were news
67

Page 39
When delving deeper into the social effects of the messages conveyed by Women's pages the following effects are evident, in the case of messages that do not result in cognitive dissonance due to socio-cultural factors.
Firstly, the messages promote social regularity. It should be stated that the socially accepted conduct that the messages provide reinforcement for, is most often, not only non-contributary to national development, but even detrimental to a great extent.
The second effect that could be identified is "Status Conferral'. In this case too most of the women who are raised into advantageous positions are not suitable for leadership in relation to national development. The status conferred to women such as beauticians, beauty queens, film stars and fashion models, merely adds to their personal profit.
Thirdly, Women's pages set the stage for discussion and decision, by deciding what women will talk about i.e., "Agenda-setting'. The social chitchat and trivial information conveyed by Women's page contribute to social values that retard development.
The fourth effect, "The Narcotizing Dysfunction' results not from a function at all, but just the opposite. We see how Women's pages bring about this effect. According to this theory, the media do their job so well that they become an end in themselves, instead of a means to an end. They charm their audiences, so that people think that they have done their duty simply by paying attention to the media. Women thus become "well informed" and spend so much time doing so, that they have hardly any time left to act on the information they have absorbed. At their worst, the media act like a drug.
The fifth effect is surveillance. This theory of the function of the media concerns their role in surveillance of the environment. In this context Women's pages provide an alert to their audience, of events that may affect their lives. In this case too, more often than not, Women's pages fail to contribute to national development.
Upto this point, the negative effects of Women's pages have been dis
cussed. However it is necessary to state that there are isolated examples of messages conveyed by Women's pages, that have a positive effect too. Women's
68

Towards a better model for Women's pages in Sri Lankan English dailies
pages also contain the strength to create compulsory effects, by direction towards their target audience.
If as Nicholas Von Hoffman, Columnist for the "Style” section of “Washington Post' says, people read the Women's page, for more than the editorial page,' why are the Women's pages considered the back water of local English dailies, scorned not only by the management, but often by the very women who produce these pages'?
Attitude of some newspapermen towards women in journalism may be cited as one reason. The number of women in journalism in Sri Lanka is extremely low. Editors are known to ignore stories about the contribution of women to development and encourage women journalists to contribute to the maintenance of the traditional Women's page. There are many editors and reporters who view all women journalists in a jocular way.
Women's page editors who want to change the content or format of their sections need the support of the management. That is something difficult for Women's page editors to acquire. More often than not the editor ignores the section and leaves it to the women he has hired, to go on working quietly within the prescribed time, catching their own errors, digging up story leads, fighting the layout battles with the printers and writing heads that fit.
Responsibility for the content of Women's pages cannot be placed solely with male editors and media planners. There are many Women's page editors who have grown up with the stock tradition and cannot break away from it. Such female traditionalists are greatly responsible for the information failure of Women's pages in relation to development.
What Ponchitta Pierce said about certain Women's page editors is an apt summery of many Women's page editors of local English dailies: 'actually they have little talent either as editors or as writers - but they have somehow landed the job....”
Before discussing a suitable model for Women's pages in local English dailies, mention should be made of audience diagnosis. In the case of these pages, audience diagnosis on a very broad scale is self-evident. The audience of the pages comprises English educated, to a great extent urban, elite.
69

Page 40
'Ze الأر
The goal of the planners of Women's pages should be to convey messages that will strengthen the participation of their target audience in national development.
With this goal in view, a model may be drafted based on the development model involving the use of appropriate technology, institutions and values. The strategy is rural based and must be implemented in the regions and hinterland, not in a few large cities. It must involve large masses of people at the grass roots level, but the participation of urban elites too cannot be undermined. In this context, women's pages could maintain a slow of information between the rural population and the urban elites, thereby harnessing the participation of their target audience in national development.
Let us now examine the structure of an appropriate model for Women's pages in local English dailies.
(a) Revolutionizing Agriculture : As Harry T. Oshima maintained, this is the key policy in a rural based labour intensive strategy for Asia. The goals are: an increase in agricultural productivity, employment, incomes, food production, wider participation in decision making; and more equal distribution of income and opportunities. To accomplish these goals, the productivity of the large majority of small peasants throughout the country must be increased.
Lawrence Kincaid and June Ock Yum illustrate the role of women in development, by discussing the rich results reaped by a group of women who banded themselves together in order to participate in development in a Korean village. The point driven by the writers of this paper is "that women can make a difference in this process (i.e., development) if they are well organised and given Support.'
Planners of Women's pages could take the cue from here. They could play a major role in promoting campaigns for the participation of their target audience in revolutionizing agriculture. Educated urban women could participate in this process by getting involved in rural development programmes. They could be drawn into participation by media messages which discuss various aspects of the subject, with special reference to the role of women.
70

Towards a better model for Women's pages in Sri Lankan English dailies
(b) Promoting Small Industries: Women's page could play a vital role in this task. Methods adopted could be the promotion of products turned out by local Small industries, instead of the customary promotion of foreign commodities. Urban women could also be persuaded to be participants in this development programme, by joining campaigns to promote sales of locally produced goods and assist individually, or as members of women's organisations, in improving the lot of rural and urban women who are engaged in Small industries.
(c) Social and Political development: Urban women should be made to realise the importance of decision making and implementation by the rural masses. Rural as well as urban women who contribute actively to social and political development should be given adequate publicity.
Instead of wasting space on discussing at length the fashions and life styles of "Colombo - 7 ladies' and foreign diplomats wives, Women's pages should discuss socio-political issues pertaining to women. The following subjects may be included: women's rights movements, the effects of urbanisation particularly on women; employment opportunities and labour welfare for women, nutrition, primary health care, women and education, with an emphasis on vocational training; housing, family planning, national integration and the necessity for social reforms.
When drafting Women's pages with development communication in mind, care should be taken to avoid the content to appear Scholarly and dull in the point of view of the target audience. Instead, the messages should be well balanced in fulfilling the tasks of the media outlined by Schramm: education, information, persuasion and entertainment.
The underlying theme of these messages should be their relevance and application to Sri Lanka, in relation to national development. Foreign sources should be used only when they serve the purpose of offering a model that could be applied to Sri Lanka. In the Gandhian tradition, the winds of foreign influence could blow upon our house, but the design should be essentially indig(CIOUS.
71

Page 41
References:
. Communication and Change: The Last Ten Years and the Next. Edited by Wilbur Schramm and Daniel Lerner. 1976. Chapter on Communication and Development by Gloria D. Feliciano P. 197
2. A Study of the Image of Women as Depicted in the Mass Media Channels in Sri Lanka. Report submitted by the Department of Mass Communication, University of Kelaniya, to the Women's Bureau of Sri Lanka, to mark the international year of women. March 8th, 1986.
3. Concepts and Issues in the Mass Media by Michael C. Emery and Ted
Smythe.
4. "The Island' newspaper 12.1.86, "Frankly Feminine" Page
5. “The Ceylon Observer” women's page 15.6.86
6. Concepts and Issues in the Mass Media by Michael C. Emery and Ted
Smythe.
7. Communication and Change: The Last Ten Years and the Next. Edited
by Wilbur Schramm and Daniel Lerner, 1976. P.24
8. Communication and Change: The Last Ten Years and the Next. Edited
by Wilbur Schramm and Daniel Lerner. 1976. P83
72

Women in the Labour Movement :
“Herstory of resistance Mangalika de Silva *
he inter-relationship of class, gender and ethnicity in the creation of a
sub proletariat of women workers has not been adequately studied in the
history of women's resistance movements in Sri Lanka. The colonial experience had a profound impact on employment opportunities for women. Colonial policy as well as prevailing cultural norms resulted in a "separating out” of women in Sri Lanka. Class and ethnic divisions altered what it meant to be a woman and what kind of social expectations guided or compelled work choices. “Women who came from the lower strata did essential if not low Status work such as tea plucking. Tamil women in the tea plantation sector have been seen as cheap labour and this perception is compounded by patriarchal assumptions that domestic labour is the exclusive responsibility of women. This has resulted in a sexual division of labour for women in which they occupy for their paid jobs and bear the brunt of the double burden - caring for their families as well as working outside the home'. (Yolanda Foster, Women and Work, Pravada/ SSA, Nov/Dec. 1996) In this paper my reading of the women's involvement in the labour movement is through the perspective of Subalterneity; women who constituted the under class socius.
* Mangalika de Silva is presently attached to the Social Scientists Association as a Researcher cun Coordinator, Gender Projects
73

Page 42
Roots of localised worker consciousness
The first signs of labour restiveness appeared among the urban working class which included literate and militant groups of workers such as printers, railway and poor workers, laundry men and carters who led the agitation against oppressive conditions.
Women's class consciousness in the struggle against exploitation and oppression received a filip with the resurgence of workers movements. This became more reinforced during the period 1893 to 1923 when several strikes of historic significance were launched across the capital city. The workers displayed remarkable resilience and militancy in organised action. The decade long period in the early twenties and the late thirties saw the astute A.E. Goonesinhe led organised workers unions gaining considerable ground in asserting their just demands. With the formation of the then two most radical left wing Marxist parties namely the LSSP in 1993 and the Communist Party in 1943, class struggles of the embattled workers acquired a political dimension unique in its historicity. The CP apart from its anti colonialist stance, stood committed to "economic development and industrialisation with the aid of the Communist block; nationalisation of foreign enterprises without compensation; wage increases, labour legislation and workers' benefits and assistance to the rural poor and tenant cultivators'. Political Parties since Independence, H.A. Peiris: 1988, Delhi, p. 14)
A.E. Buultjens pioneered the protest movement with the formation of the first trade union in Ceylon 1893, (the Ceylon Printers Union) after a strike of printers at Cave & Co. (Jayawardena: The Origins of the Left Movement, 1978, Sanjiva Books. P4-5 "Thereafter, numerous unorganised, spontaneous Strikes and a few organised strikes of organised laundrymen (1986), Carters (1906) railway workers (1912) and harbour and railway workers (1920)' reflected the growing consciousness of the possibilities of joint action against rampant exploitation and multiple victimisation.
LSSP supported the first organised union in 1932 which was the Wellawatte Mill Workers Union. The LSSP in 1932 was ideologically committed to ensuring "socialisation of the means of production and distribution; abolition of Social and economic inequality; the elimination of oppression arising from differences of class, caste, race, creed of sex". (H.A.Peiris, 1988 p. 19)
74

Women in the Labour Movement : “Herstory of resistance
Revolutionising the working class
Capitalism in Ceylon was plantation centered. With the growth of plantation capitalism, there emerged crucial cracks in the class composition and political superStructure of the country. Several important political trends were visible. A resurgent movement for political reform and the labour movement which encompassed strikes, workers agitation and the formation of trade unions surfaced. As Kumari Jayawardena has identified the "heavy concentration of the economy on export earnings of three crops (tea, rubber and coconut led to the neglect of the rest of the economy resulting in a virtual absence of a class of local industrialists and traders. Major industries were owned by foreign nationals. Local enterpreneurs were associated with Small Scale manufacture of matches, becdi, furniture and brushes. Domestic trade was heavily British owned. However, the development of transport and urban workshops ancillary to the plantation economy gave rise to the nucleus of urban skilled and Semi skilled wage labour'. Jayawardena: Origins of the Left Movement. P3A new form of employer worker relationship was developed. A segment of this working class “sought to improve their status through organisation and joint action. During the fifty year period between 1880 to 1930, the workers' agitation for trade union rights was linked with the movement for constitutional reforms led by the bourgeoisie', ibid. p.4
Organising for change
In the face of fierce resistance to granting of universal Suffrage to women, vehemently opposed by nationalist reformist Ponnambalam Ramanadan, A.E. Goonesinhe advocated vote for women on the same basis as for men. He saw the extension of the vote to women as a "natural and inevitable next step to manhood Suffrage. His argument was that just as much as illiterate male labourers were intelligent enough to exercise the vote wisely, the Women too could be expected to act in the same way” Women in our Legislature, 1931 - 77, Chitra Wijesekera, 1995. Vishva Lekha. P. 14)
The growth of capitalism changed the old social order and gave birth to new classes and new strata. Under Such circumstances, to foreign and local capitalists and landowners, women were the cheapest source of labour for plantations, agriculture and industry. “While it is true that women had toiled in the fields and plantations and domestic industries in the pre capitalist phase, it was
75

Page 43
'മd %لأر
with the development of capitalism in a colonial or semi colonial context, that they were to become available as potentially the largest and cheapest reserve army of labour. Women's labour was therefore very important to local and foreign capitalists: traditions and practices which restricted women's mobility or enforced their seclusion were thus detrimental to capitalism in its search for cheap free’ labour. Women's labour was also crucial in the plantation sector (tea, rubber, coconut, sugar etc.) and in farm and domestic agriculture in these countries”. F & N: p.9 “While the women of the peasantry and working class were being proletarianized, those of the bourgeoisie were trained to accept new social roles in conforming with the merging bourgeois ideology of the period. The women's struggles for reforms were generally focused on issues of equality for women within the legal process, the removal of discriminatory practices, the right to the vote, education and property and the right of women to enter the professions and politics”. (ibid.: p.9)
Education as a tool of emancipation; Intersections of class and gender
Education was meant to mould women into domesticity. Nationalist reformers appealed to upper middle class women to be good wives/mothers while men aspired for upward Social mobility in the colonial setting. Education, a privilege, was denied to the under classes in the peripheries. They constituted the serving class. They formed the backbone of the colonial economy. "The indigenous bourgeoisie had no intention of applying the concepts of natural rights, liberty, equality and self determination of the masses of women or to the workers'. A.E. Buultjens, influenced by both British socialism and the trade union movement extended his commitment to female education.
"The rise in women's consciousness brought about by education was invested primarily into struggles for political and franchise rights”. (F & N 1986. p. 130 Women's Franchise Union formed in 1927 publicly agitated for Voting rights for women. The campaign for Suffrage rights for women culminated in the granting of the female vote through Donoughmore Constitutional Reforms of 1931.
76

Women in the Labour Movement : “Herstory of resistance
у
Struggles of working women
"In the 19th century, the most exploited group of workers in Sri Lanka was the large semi proletariat of women workers on the tea and rubber plantations. Thc wage rates for estate women were the lowest in the island and their general mortality, maternal mortality, infant mortality rates on plantations are today the highest and the literacy and education rates the lowest in the country”. (Jayawardena F & N p. 13 || ||
There was a sizeable unskilled workforce of women employed in factories; coconut fibre mills, tea plucking and graphite Sorting Jayawardena F & N p. 131 There was an underpaid and overworked group of domestic servants who worked as Semi slaves. Peasant women worked as wage labour, share croppers or family labour in the country-side and were often at the receiving end of exploitative local owners. Women came to the forefront of trade union activism during the general strike of 1923. Jayawardena p.131 A.E.Goonesinhe was backed by militant red dress clad urban working class women in every strike, procession, demonstration and May Day rallies organised by the Ceylon Labour Union. They took an active interest in all working class struggles to win their democratic rights.
Militant women in the movement Isabella Hamy
An early firebrand at the height of Goonesinhe led worker struggles was Emaliya Hamy of Hunupitiya, a member of the women's volunteer corps. Other pioneer worker activists included Pavistina Hamy, who worked at Hayleys and Emmy Nona, a worker at Harison and Crosfield, both British firms. As Kumari Jayawardena writes, "the best known of these working class militants, however was Isabella Hamy of Wanathamulla, Colombo, a woman who fought for the rights of the working class at the height of colonial rule, when trade unions were illegal and exceptional courage was needed to join worker agitations'. Jayawardena p. 32 Isabella Hamy was an articulate public speaker who militantly spearheaded a trade union in the 1920s. She won the admiration of Union leader A.E. Goonesinhe. During the 1929 Tramcar strike, she organised demonstrations, participated in numerous meetings and processions and was at the forefront of this famous struggle”. (ibid.: p.133 Her rise to popularity was Swift, and she was a formidable political person who rigorously canvassed for

Page 44
.á لار.
A.E. Goonesinhe during the State Council elections of 1931. Tenacious and full of passionate zeal, she won the honour of being called "Captain' by her comrades.
Ponsinahamy
Ediraweera Arachchige Ponsinahamy was a notable figure in the anti imperialist, anticolonial worker struggles who fought against oppressive institutional measures adopted during the time of the British Colonialism. Growing up in Mariyakade, Ponsinahamy became acutely aware of the degrading work conditions women were subjected to and challenged authorities to rectify these anomalies. She was daring, tough and aggressive in the face of arbitrary abuse of power. She received no formal education. Ponsinahamy was clear in vision, eloquent in speech and fierce and utterly unafraid in battle. During the Tramcar strike she lay across the path preventing the trams from running. She was instrumental in bringing victory to the strike in 1947 with its demands for higher wages and lunch intervals. Ponsinahamy, Mala Dassanayake, Voice of Women, March 1981. p. 12) Often alluded to as “Veera Ponsinahamy, she was held in high esteem and exerted a great deal of dignity and respect.
As a crusader of workers rights, she championed the cause of working class women in their struggle for equal wages and improved working conditions. She took pride in the honour of being the only woman represented in the delegation sent to Russia to commemorate International Workers Day in 1967. ibid.: p. 13 She faced police harassment and violence during the general strike of 1923 and became a victim of state persecution when she was sentenced to three months imprisonment in 1947. On Independence Day, she was granted an amnesty and she was free along with others in the movement with her, once again to enter the political arena.
Maimoon
Lean yet dynamic and daring, Maimoon was an active left veering Muslim woman who developed ideological leanings towards the Communist Party. At the height of the volatile issue of the "Sinhala Policy” espoused so assiduously by the ultra nationalist left of centre SLFP led by SWRD, Communist Party, loyalist who stood in the audience with others where a meeting in support of the CP stand was being held. H.G.S. Rathnaweera was addressing the
78

Women in the Labour Movement : “Herstory of resistance
crowd, when a Buddhist priest, obviously provoked, started heckling the speaker. Maimoon walked upto him, held him by his robe and pulled him down admonishing him to remain silent. Outraged by this provocation, and humiliated by this offensive behaviour, angry hostile mobs stormed the meeting, assaulting the activists and Supporters of the CP. Personal Communication, Kumarasiri p, 1996 Originally from Wekanda, Slave Island, she earned a reputation for fearlessness, aggressiveness and eloquence in speech. As a regular platform Speaker, she attracted large crowds. She had received no education but developed Oratorical skills and often thrilled the crowds with wit and humour. Majority of women like Maimoon had no formal training in schools. They were not of School going age when free education was being introduced in the late 40s. Education was limited to upper class/caste sections.
The quest for class liberation historically had filtered through the existing patriarchal Social structures. Worker demands inevitably met with resistance and in other areas, Success was mainly due to these limitations. The workerS movement could not move away from its essentialist focus on political economy to the larger issues of social marginalisation, violence and even exploitation. The economic parameters were severely restricted under colonialism and with the womcn's movement could be identified class hierarchies that denied working class women the space to influence if articulate hitherto unaddressed Social concerns. A major responsibility lay with the dominant segment of negotiating power within a hegemonic patriarchy.
Graphite mine workers
With the development of several local industries, graphite mines, cinnamon plantations, agriculture, coir industry, FTZs, garment factories and migrant domestic Labor, there flourished a significant semi proletariat labour force of women exceeding 50% of the population.
Women were initially recruited into graphite industry owned by wealthy male industrialists for 'soft job categories as they were perceived as physically weak and therefore possessed an inherent incapacity for performing heavy tasks. Women Graphite Workers, Rohini Weerasinghe, Voice of women, Vol 3 issue 3, 1988, p.5. Here too, the traditional gender ideology has penetrated into existing structures which classified women as soft and nimble fingered who willingly worked under the care and protection of men outside the
79

Page 45
'Z هلال
mines whilst men were engaged in heavy arduous work under dangerous conditions inside the mines. The women in the mine sector were a highly politically conscious group radicalised by left activism in the early 50s and politicised through their involvement in worker agitations and class struggles. They were among the best organised groups in the 1953 Hartal and displayed courage and militancy in the face of repression and frontal attack. (ibid: p.7 Their committed involvement in the successive campaigns paid dividends when the industry was freed from state control in 1970, ibid.: p.7. They formed the Sri Lanka Graphite Workers Union soon after the peoplisation of the industry. The subsequent seething labour discontent, with the opening of the market economy in the late seventies, burst forth in a series of disturbances, sudden work stoppages, strikes and pickets, against the soaring cost of living, closing down of factories and privatisation. They formed a network of unions and extended their solidarity and Support for just demands. The 1986 nurses strike was strongly backed by the graphite workers conviction that without class struggle, there cannot be full women's emancipation and vice versa.
The organised female labour force concentrated in the FTZs, posed problems which have been of a different nature. "Neither the state nor the employers were supportive of trade unions and erected numerous obstacles including the harassment and threats to life of trade union activists. The mainly female workforce were directly from the rural areas and did not conceive themselves as being long term employment'. Trade Unions in an Open Economy: State, labour and law in Sri Lanka 77-94. SSA document p.2 "Their particular needs and grievances were largely centered around gender specific issues and the male dominated trade unions have to date been unable to alter their habits and practices to accommodate themselves to this new and important sector of the industrial proletariat”. [ibid: p.2)
Women in the FTZs
Women in FTZs have signed employer contracts which violate the very existing Sri Lankan labour law and human, civil and constitutional rights: The logic behind the recruitment of rural female labour was simple: they have relatively little or no experience of paid employment. "Delivered from the arm of village patriarchy into the waiting embrace of industrial patriarchy, these women are seen as the ideal work force for Sri Lanka's low wage garment and manu
80

Women in the Labour Movement : “Herstory of resistance
facturing industries'. Women in the FTZs of Sri Lanka. Janice Fine & Mathew Howard in Dollars and Sense, Nov./Dec 1995 No.202 p.26
Despite their high level of education, women are not encouraged to contribute their ideas into the production process. They demand improvements in “transportation, safety conditions, raises, appropriate pay for overtime and end of compulsory overtime". Iibid.: p.40. At night, often, they are compelled to travel in the dark (no street lights) to return to their quarters after work. “Kalape Api', a community based direct action organisation has documented the instances of sexual harassment, rape and other Sexual crimes committed against these women. They have no right to unionise' although wonnen have taken part in some form of collective action'. (ibid.: p.40) Their demands for better housing, work conditions and exploitation and violence free environment are channelled through social service organisations and community NGOs. There has been much community organising in and around FTZs to attract an active participation of workers in labour activism with the view of conscientising them on the need to build a mass Labor movement of workers. The Unions that act as catalysts or mobilisers for social change in the FTZs in the form of CMU, ITGMU and the CFTU are all male entities which illustrate the hegemony of the patriarchal capitalist power structures entrenched in the industrial Zones. Not only is there gross under-representation of women at decision making or leadership levels, there is resistance if little encouragement for political mobilisation of these workers to politically engage in transforming oppressive work StTLC tuICS.
Tea Plantations
Women constitute over 50% of the workforce in the plantation, the biggest recorded ever in any sector. Women have been especially encouraged for lower paid jobs as tea pluckers because they were seen as an unintelligent and abundant source of cheap Labor, Women are faced with domination from their husbands, fathers and brothers and even their male overseers, kanganies and planters in the field. Infrastructure has been anything but satisfactory. They live in line rooms, so the boundaries of private space have been erased out. There is no primary health care system. Hospitals are located miles away from their residential quarters. Schools are ill equipped so that educational/literacy levels have remained abysmally poor. "Even today women are not given an equal wage for the same work that man is given. While the margin had nar
81

Page 46
M2 Ze'
rowed from a 25% wage difference 150 years ago to 10% in the 1980s, blatant discrimination still exists. The wage is also dependent on the number of days worked during the month. With the privatisation of the tea industry, the amount of work available is not always guaranteed". Minimum wage is paid depending on the number of tea leaves plucked. Women have no control over their resources and income as men in their Social roles as fathers, brothers or husbands exercise authority over their remittances. Illiteracy stands at 80% in the plantations which is arguably the highest recorded for any sector. Gender and the Tea Women, Danielle Atkin, Voice of Women, March 1995, p.8)
The plantation runs as a pattern of Social control. "The colonists designed the plantations on the basis of hierarchies which made use of other pattern of social control embedded at different levels of the Social field, such as caste'. Patriarchy in the Plantations, unpublished paper: Yolanda Foster, SSA, 1996, p.4 The system of Labor recruitment based on the head Kangani (HK) is a classic case of this authority and centralisation of male power. Women workers often do not have time to liaise with trade unions, to stop the erosion of workers benefits to get women into power.
The totalising and totalitarian CWC is an all male enclave where women continue to serve male interests. Given the dynamics of the oversupply of Labor, women's resistance to change gender power relations, transform existing hierarchies and develop alternative human and equitable development model will echo through their union activism. All in all, the social construction of gender, sexual division of Labor and the unequal power relations have been significant contributory factors to the low participation of women.
Conclusion
The right to unionise and engage in organised democratic action has, been the most contentious and intractable issue facing the trade union movement today. The democratic right to participate in worker agitations and make representations at the national level has been denied to three most vital sectors of the economy. Migrant workers, the agriculture sector and women in industrial zones. The public Services Nurses Union is led by a member of the Buddhist clergy. Migrant and peasant women have been worse affected as often their relative powerlessness has been perceived as a drawback which deterred them from addressing issues specific to them; from issues of sexual violence, to
82

Women in the Labour Movement: “Herstory of resistance
worker employer relations, the right of preview of contract of minimum wages, payment of cash at regular intervals, maximum hours of work and Social security and welfare benefits. While the state proclaims as the moral protector and guardian of the citizens, its commitment to upholding domestic legislational norms has been blatantly discriminatory. The Workers' Charter has recognised in principle the right to form unions and frames progressive legislation to strengthen workers rights. What is indeed necessary is legislation which empowers women, legislation that can radically transform work relations, a reality that still eludes women.
83

Page 47
M2 á.
Human Rights Education and Empowerment of Women
Selvy Thiruchandran
his paper is divided into three parts. I see three major issues embedded in the title and in fact the three parts would delineate the connections between the three dimensional contours of the title of this paper.
The first part will identify the nature and typology of the human rights that we envisage and hope to incorporate into the package of education. As we may note the concept of human rights is not a value neutral terminology. The second part deals with the connections or rather the linkages of such a package with the contemporary situation in the plantation sector, where such rights are violated. This process will simultaneously identify the inadequacy of the structures that operate there. Some of these structures should have entrenched a human rights orientation and ideology in their agenda but have failed to do so. Political parties and Trade Unions are an example. Patriarchy which usually operates in the private realm of family and religion has been introduced into political structures resulting in a concept of public patriarchy which needs to be analysed.
* This paper was presented at a Seminar on "Human Rights in the Plantation Sector", held on the 17th January '97, under the sponsorship of the Social Scientists Association of Sri Lanka.
84

Human Rights Education and Empowerment of Women
The third section takes up a relevant question whether, an education in human rights, will necessarily lead to empowerment. As, social activists, we are not merely content with imparting an education, but are committed to finding out whether it can be tied up with an empowerment process because empowerment is what we are concerned with, in the final analysis.
What we mean by human rights has to be conceptualised initially, as the discourse of human rights, has both legal and social implications. Its political implications can also have an international dimension. Human rights in my opinion should envisage a totality of a situation which should guarantee that human beings can live with dignity and honour, enjoying a status of equality as human beings with no special prerogatives assigned or demarcated to persons of a particular breed, caste, class, ethnicity or gender or of particular religious belonging. Gender has been out of the agenda of human rights for long and women's rights as human rights is a phraseology admitted of late into the discourse of human rights by the feminist intervention.
It is fashionable nowadays to often talk of cultural relativism within the discourse of human rights. By cultural relativism is meant that due to a particular, cultural belonging a community can lay claims to special prerogatives as exclusive rights granted to them either by the religion or culture to which they belong. Logically cultural relativism works against the principle of universality of human rights. It leads to a view that human rights can mean different things to different people. Those who argue for cultural relativism are seen as against the principle of universality of human rights. Such a contention is flawed. The argument is misplaced and leads to much misunderstanding. While stressing that there is validity in the arguments for the universality of human rights, I want to argue that the universality of human rights, should not be placed dichotomously against cultural relativism to negate or belittle the former. And while arguing for the universality of human rights, cultural rights that qualify to be within the accepted code of ethics, which do not necessarily lead to discrimination and violations of a grave nature should be viewed with sympathy.ie., While the rights of a community to sing "bhajans' early in the morning at 4.00 on the Streets, disturbing others, who do not believe in such practices and who are peacefully asleep, and the rights of a community to transmit through the loud speakers their pious prayers which disturb the other citizens who want a noise-free peace in late evenings, have to be conceded and recognised as cultural rights and religious rites. Such disturbances have to be couched within a framework of tolerance and duties and obligations to fellow citizens, and not as
85

Page 48
മe الأر
violations of ones rights by others. But all cultural practices cannot be tolerated. Neither can they be considered duties by others.
Female infanticide and burning the dharmic wife alive on the funeral pyre of the lord husband cannot be considered cultural rights of a community or as religious rites. They are aimed at specifically harming, hurting, destroying and discriminating against one set of human beings who belong to a particular gender.
The exercise of cultural rights of one community or group has to be tolerated and respected. Whether one believes in them or not is another matter. In today's context, in the present era of identity crisis where identity is asserted on religious or ethnic belonging a public conscience sensitive to its civic responsibilities is expected in the exercise of human rights.
The second point of discussion that one usually confronts within the discourse of human rights is that the concept Smacks of Euro-centric individualism and does not take into consideration the duties performed mutually and obligations fulfilled in return. Such types of social interaction with duties and obligation is communitarian and includes the notion of service. It is argued further that this is peculiar to Asians and that Asians have a particular value system which is disregarded by the demand for human rights. Such claims can be contested on various grounds. The cultural patterns of a society do not remain static if at all such a glorious value system did exist. Asian societies were built on very clear hierarchical inegalitarian orders that caste and gender rules are/were followed meticulously in South Asian societies as duties performed according to some dharma based an one's birth and gender belonging. This is an example where no consideration was given to the hunian rights that value equality and dignity. The hierarchical and oppressive socio- economic relations are now challenged under the rubric of human rights.
Besides, this is an ethno-centric view. In the west one can come across images, visions and realities of the milk of human kindness which have argued for collective duties and community well being. Western history and literature - poems, novels, plays are full of such values which we call "Asian'. Contrariwise in Asia there had been so much of individualism - tyranny of absolute monarchies, religious persecution and sufferings, mutual enmity, competition and jealousy have resulted in wars.
86

Human Rights Education and Empowerment of Women
How truthful are we to speak of Asian values as glorious and noble'? The suicide rate is highest in Sri Lanka. Violence is now rated as the highest taking the place of Nigeria. Violence has been our history for the last three or four decades.
Human rights should mean a totality of rights, civil political social and economic. Rights cannot be fragmented. You cannot have Some rights and not the other. Rights cannot be prioritised as one over the other. We classify them and categorise them as civil, political, Social and economic with universal standards taking into account the limited use of the concept of cultural relativism. The idea of basic needs, hunger-free existence with a roof over one's head needs to be incorporated as basic rights into the arena of human rights. Other rights, political and civil are meaningless without the basic rights.
This takes us into another terrain of the debate in human rights - the Marxist position on Human rights. It is generally assumed that Marx treated human rights as irrelevant to the socio political process. Marxism which is a comprehensive world view has advocated among other things, a free society where everyone's needs are satisfied (Basic needs) and where hierarchical social grading Such as those based on class will disappear (equality). Implicitly it means that there is no need to talk of Human Rights when that state of equality is achieved. His conviction that Human Rights are “ideological nonsense' and 'absolute verbal rubbish' stems from another conviction. He was convinced perhaps wrongly, that the so called civil rights and political rights, the preoccupation of the bourgeoisie at that time in history, are inadequate in So far as the Social and economic rights of the working class are not secured.
While asserting that human rights cannot be compartmentalised or prioritised as one right to be more meaningful than the other, it has to be admitted that the hourgeoisie is just not one social bloc of people. They are divided on ethnic religious and gender lines. Their life experiences vary to the point of being marginalised, discriminated and oppressed as groups and communities, and treated as the other to the point of de-citizenising them. They are at times denied their rights as citizens and they need to assert not only their civil and political rights, but the other rights as well. A girl child of any class, can be denied her education. Amniocentesis is performed across classes to determine the sex of the unborn baby. Dowry deaths of brides take place even among the upper classes. Hegemonic social, political, ethnic, or religious and
87

Page 49
M2 幼
gendergroups among the bourgeoisie can oppress other groups within the same bourgeoisie who are less powerful. Historically and contextually the positions shift thereby creating conditions of oppression across classes and gender. Human Rights, then should extend beyond class categories and within the framework of democracy.
The concept of social democracy is relevant in the discussion of human rights as it is within this framework that we draw up more concretely an overall agenda for the operationalisation of human rights. Social democracy in terms of what it means has a history over the past century and a half. Historically it has the meaning of a coalition of forces of the bourgeoisie and the working class. However, with the idea of a struggle to attain political democracy which includes the notions of elected bodies, the principle of representation with the right to vote came finally to be linked, so that the ultimate aim is to extend the principle of democracy to social life. This characteristically included the organisation of production as well. This it was hoped, works against class domination and works eventually towards a general Social emancipation, implying in Marxist terminology, human emancipation. The concept of human rights has to be linked to human emancipation which is not merely the emancipation of the working classes but humanity as a whole with an emphatic insistence on the doctrinal value of democracy with its elected bodies and representative institutions. Socialism and democracy have been conceptually coordinated to work towards a comprehensive plan of human emancipation. This is my contention of what human rights should mean and how its principle should be enshrined in the contents of the human rights education, with an emphasis on civic responsibility.
Last though not the least is the gender sensitivity that is needed both in its contents and in the consciousness of the instructors. Despite upper class belonging and belonging to the hegemonic bloc, women become marginalised because of the overall patriarchal regime which is pervasive. The plantation women in addition to the Suffering imposed on them because of their class and ethnic belonging are further marginalised by their own institutions, values and norms which are constructed on patriarchy. They suffer a triple burden which needs to be identified in the educative process. (The burden of traditions, the burden of the working life and the burden of the household).
88

Human Rights Education and Empowerment of Women
Human Rights Violations and Women in the Plantation Sector.
This section deals with very specific situations of human rights violation that are peculiar to women on the plantation sector. It is hoped that the contents of the education package focuses on their real problems and on the reality of the situation. This analysis is done from a two-way methodology which is partly empirical and partly from information collected from the media. On recent visits (in May/June 1996) to the plantations in connection with a research programme I met members of a few NGOs, and fifteen working women. The Tamil Daily, Virakesari has a weekly page for the plantations where socio-political issues are taken up, for analysis by social activists and others from the plantations. The plantation Tamils call themselves as Malaiyaha Tamilar - Tamils from the Hills. My discussion is based both from an analysis of the columns of Virakesari from January 1996 to December 1996 and the interviews I had with women and members of the NGOs. The life experiences of the women are discussed here with a view to identify the human rights violations so that they will become the focal points of the human rights education programme, which could work for socio-political transformation.
Despite years of Trade Unionism and political activities by their leaders, the plantation workers still remain one of the most backward and oppressed communities in Sri Lanka. The landless proletarianised workers are poor, with many socioeconomical deprivations where their basic rights to life are denied. The low wage structure at the plantations compares unfavourably with those of the wage structures elsewhere. Poverty is pervasive despite the fact that many members of the family are in the production process and this factor obscures the individual low wages which is reduced to mere subsistence level.
Amidst these general patterns of exploitations are the gender specific exploitation to which the women are subjected. While saying this it is not implied that men are at a advantageous position to be envied at. It is merely to show the different levels, so that conscientization process can take note of the disparity levels and focus the contents on the issues that need to be addressed.
Exploitation of Women
The over poundage system allows women to earn more money than men. They do so, at a heavy loss. The greater effort is compensated by lower
89

Page 50
άπει لإر
rates which becomes lower and lower, the more they work (Hollup : 1994 : 110). Women workers do not receive the same supplement to their monthly income called free feeding or the allowances which is added to the male wages. It is reported in the Virakesari of 18.01. 1996 that in a Pussellawa estate the management had insisted that women workers should work from 7.30 a.m. to 5.00 p.m. on a compulsory basis. As a result the mothers could not breast feed the children. Six husbands who objected to the extra working hours were suspended from work.
The working conditions of the women pluckers are below the expected minimum standards. They are not offered any protective kits against the climate and the climbing blood sucking leeches. With bare feet and bare hands and wearing clothes that do not protect them against the chill and cold, the women climb the heights of the hills, daily earning the much needed foreign exchange for our country. The extensive use of insecticides affect the women. The women inhale them in addition to the tea dust from the factories, when they are at work in the factories. They live in a continuous state of ill health. Malnutrition, miscarriages, lack of resistance to infections are complaints that I heard from the women. Creche facilities and day care centres are exceptions than the rule.
Engels has said that there is no stimulus to assert male domination in the absence of property among the proletarian families. He was also of the opinion that due to the position of women as “bread winners' of the home, male domination has lost its foundation (Engels 1884:72) However, in the plantation homes Engels has been proved wrong. Without property and with the status of the bread winner, the plantation women (who sometimes earn more than their husbands) still suffer male domination and inequality in the family. In addition to being plantation proletarians the women play the role of domestic proletarians in the home doing hazardous tasks such as drawing water, grinding, pounding, cooking and cleaning. A radical transformation within the house where men and women share the household domestic labour and child care may be possible, if gender sensitive human rights education is imparted.
Violence Against Women
Violence against women takes two forms in the estates. Women complained of sexual harassment from the kanganis. The sexual harassment takes
90

Human Rights Education and Empowerment of Women
many forms from eve-teasing to verbal indictments causing psychological impact which is demeaning to the self dignity of the women. The other type is the violent intrusion on the autonomy of their bodies by physical violations ranging from holding hands, brushing against them, coming in close proximity. The most violent form of this is rape.
Violence by one's own people usually takes place within the family. Domestic violence by husbands is often explained by the women as due to alcoholism, irritation and frustration caused by lack of money. Excuses are explained as reasons by these women due to a particular type of socialisation that they have gone through. Frustrations in life and poverty are both suffered by women and men alike, but their expressions in the form of violence becomes a male monopoly. The Women are sensitive to the violations that men inflict both on their psychology and on their bodies. However, there is a tendency to accept male violence as natural. That "women's rights are human rights," needs to be instilled both into the psyche of men and women.
Health Status
The health status of women at the plantation is the result of the lack of Sufficient infrastructural needs that go to make minimum standard of basic needs. Long working hours under harsh conditions, problems of food both in quantity and quality, the double burden of doing the household chores in illventilated kitchens where they inhale the Smoke are part of their daily life experiences. These, in fact are the causes for malnutrition. They have particular illnesses commonly identified as caused by specific living conditions. Goitre due to lack of iodine in food, under weight babies, blindness and illness connected with the liver are some of these. The health services in these areas have serious drawbacks.
The expectant mother is transported, often when she has labour pains in lorries that are used to transport tea leaves. After the delivery both the mother and the baby are also transported in the same lorry. Women who undergo sterilisation after a three hour operation are also transported in the same lorries. In the name of free medical service, grave injustice is being done to these women. These inhuman practices have often been brought to the notice of the women's groups and NGOs but up to now no remedial action has been undertaken. It appears that the health service has differential standards for the plantation Sector. The hospitals and the medical care given at these centres are not compa
91.

Page 51
rable to national standards. That the national standards are not maintained at these medical institutions is a human rights violation and we should take it seriously as it is within the citizenship rights.
Social and Civil Standards
Social and civil standards that citizens enjoy in a state are the reflections of the rights to which they have access. Sectoral imbalances in a country are indeed the reflections of the denial of the rights to some sections on ethnic, religious, gender or on other grounds. While the all island literacy rate is 87.2 and that of the rural areas 85.4, the estate rate is a mere 40% and the female literacy rate is much lower than that.
Acquiring birth certificates, death certificates, national identity cards, food stamps are indeed problems for many of them. This drives them to a dependency status on a few others who are literate. Birth of children are often referred to as male and female when their births are registered. The identity which the name gives are denied to these children. It is very easy to explain their illiteracy as due to apathy or disinterestedness of this class of people. But this is beyond a simple rationalisation, it needs to be couched in an argument of deprivations to which one should hold the state responsible. Illiteracy, and bureaucratic inconsideration prevent the realisation of their full citizenship.
Child labour, both in the estates and in the urban centres as domestic servants thus becomes the means by which the parents earn their supplementary income. To speak of preventing child labour is not to solve the problem. Child labour has to be linked to economic conditions. Violations of some rights lead to violation of a Series of other rights.
Feminised Roles, Male Domination and Trade Unions
Women do appear in the trade unions in numbers as nominees of men or party members. They are excluded from management positions, authority and power are generally male preserves and not delegated to women. They are usually excluded from the decision making bodies. Neither is their involvement in financial management accepted. Women's issues, such as the need for creches and women's wages are hardly discussed in committee meetings. Women
92.

Human Rights Education and Empowerment of Women
are generally assigned the task of buying gifts for social and religious functions and attending age attainment ceremonies and weddings. Another complaint against the trade unions which is equally valid is the undemocratic manner in which trade unions function. Leadership is the monopoly of a few. It is alleged that political parties intervene and destroy the unity of the members and the interests of the trade unions are sacrificed for party interests. There are too many break away groups and this has weakened the Trade Union movement.
Interestingly even the NGOs who work in the plantations are also blamed. The ad hoc and insufficient benefits offered by the NGOs, it is said have weakened the militancy of the trade union movement and often issues that need to become agendas for the unions are dealt with by the NGOs. (Navamani 9. 0.96, Yathawan). These are very similar to the assertions made by James Petras in his article on Social Scientists, NGOs and Intellectuals in the Lanka Guardian (Vol. 19. No. 1 October 1996). The rights of the members of Trade Unions need to be asserted, and democratic principles should be introduced by the members. It is in fact reported that the workers are losing confidence and faith in the Trade Unions. (30.06.96, 13.10.96 Virakesari). Women's feminised roles have to be reverted to gender neutral functions with greater responsibilities, powers, and authority towards a meaningful transformation.
Job Agencies and Jobs abroad
There have been many cases of duplicity reported in the media. Unsuspecting women have been literally taken for ride to Colombo, after having paid large sums of money. The women have pawned their jewellery and got loans to pay these largesums of money. After having been housed in Colombo for a few days, they go home realising belatedly that they have indeed been duped. The agents have disappeared. (Namuoya, Kadugan nu wa 04.05.96, 19.06.96Virakesari). It has been reported that most of these women are illiterate and could not even sign their names.
The right to education having been denied these women, they have no recourse to judicial procedures to sue the job agents.
93

Page 52
άτι الأر
Human Rights Education to Empower Women
A programme on human rights education should start on a note of gender Sensitisation. By gender Sensitisation is meant a process of raising the consciousness of men and women towards patriarchal structures, institutions, and ideology. Patriarchy simply means male exercise of power, authority, and domination in the socio-economic political and family life, to the extent of going to violent means to Satisfy the ends of male domination. Deconstructing and deciphering this whole process is to gender sensitise the actors. Industrial wage labour to domestic unpaid labour, trade unions to political parties, medical Service to State bureaucracy, temples to churches are governed by the rules of patriarchy. Equal rights, equal opportunities, freedom of movement, freedom of expression and the right to dissent are denied to women in their social and family life. Hence women's rights should be viewed as human rights in the programme of education before one Speaks of their political and other rights. It may be noted that both men and women should be educated in human rights and exercising these rights judiciously should be the ultimate goal. Efforts should be made to instill a sense of democracy in the internal management of the trade unions. Norms and practices should improve in trade unions, by delegating authority and rotating the leadership.
One is of course sceptical about the results, when the history of the plantation is taken into consideration. There has been activists, NGOS, trade unions and political parties exclusively engaged with the plantation people - the results are disappointing. The reason perhaps is that a radical transformation in the basic and fundamental structure of exploitation has not taken place. Poverty is one major problem and eradication of poverty for both men women should pave the way for the realisation of other rights.
By educating the people are we trying to conscientize them to rise against
the system in a revolutionary manner'? If the answer is in the affirmative then human rights education will have some meaning.
94

Human Rights Education and Empowerment of Women
References:
Engels Frederick : 1972, The Origin of the Female, Private Property and the
State, Progress Pittsburgh Moscow
Gay, Peter: 1952, The Dilemma of Democratic Socialism
Przeworski, Adam 1980, "Social Democracy as a Historical Phenomenon" in
Praxis International l, No I 1981.
Hollup: 1994, Bonded Labour, Costs and Cultural Identity among Tamil Plantation Workers in Sri Lanka, Charles Subasinghe and Sons Sri Lanka.
Petras, James : 1996, Social Scientists, NGOs and intellectuals in Lanka Guard
ian Vol. 19. No. 1 1 October 15th.
Newspapers
Virakesari 18.01.96, 28.01.96, 30.0696, 13.10.96, 19.09.96
Nawamani 9. 10.96
95

Page 53
M2 4.
Agate Days
Eva Rana weera *
Days They were lovely as when we looked from the eastern mountain top and the valley below green as green as my youth binding us two with a silver band on fingers long and young
winding
along our bodies
and flowing to the ocean
the green blue of the water
keeping us one
in the ocean reflected
the river of my valley
following us here
to make us one.
* Ms. Ranaweera a novelist, a poet and a dramalist. She was the Sri Lankan representative in the Afro Asian People's Solidarity Organisation for fourteen years. Ms. Eva Ranaweera is presentely attached to Voice of Women.
96

Agate Days
Days counted in the dark of the night when no one was in sight carefully taken out
one bead at a time
navagune, the nine virtues
beaded on to one necklace
to be chanted one hundred and eight times
I need more
my days are eastern mountain high. So I go on again and again in a crooked circle In the dark of the night
take one at a time
days
kept in the silver casket
threaded on a silver string navagune with agates as mother's was as given by him
instead of the thali.
I count
my back to the wall this night wind is very cold in the camp taking them out
one at a time
sure as sure can be
put them back worse may be the time to come
hoarding in war and love
97

Page 54
/% Ze
is customary and a survival planwhen specially we had been under the mango treethe very special ones come out for the shade.
and eyes
and hands
etching lovely days mango friends under mango trees - grooming to be leaders
standing under the mango tree.
how black can eyes be? the eyes I know measured by yard sticks and mileage and I swim in the inky ocean hold the bicycle
for two
ridden by one
carriage for the lighter one
the sun and the moon did not smile under the silent tree they watch me, the billion leaves
but I know
we were the sun and the moon under the mango tree Just to cross the road from King George's Hall to College House jump on the bicycle, the pillion is laden
"feather light" he says.
Stare into my eyes what you see
Stare into your eyes what I see
all the mango friends were waiting
98

Agate Days
for the misty fragrant future
swimming in the inky Ocean.
Mother said you fool, father said what a shame where is the race and where the ancestor's religion? For two long days there was crying Mother said eat eat child I also did this thing under the bo tree where the leaves sing the sweetest the sweetest I have heard, making bodi-pujas It will pass my dear there will remain only a sting it takes a long long time to get that out, but you sing like the rest, so you can keep the Sting
ԱmSeem
it's precious. It turns into a scar slowly No mother No. I want him; no sting, no scar Rivers are running in the house and the house is wet such a small cye can make long long rivers in two days
Mother I die; my friend, my mother, show him to me mother, you tell him no'?
Who's at the window knocking on my heart? Silence is golden friend don't make a start before the cock crows that's what the magic man said. fly away fly away on the wing
mango friends are mango friends and mango friends will be old friends hot is his hand on the bicycle seat rattling like mad when speeding away the cops chasing
two is too much, one on the bicycle
99

Page 55
á لار
run heart run heart cops are chasing and the friendly night is dark couldn't wish for better
just escape the big cop eyes run heart run can't he see it's only me, we are one on the rattly rattly thing
looking for a hide place just hold the hand the hard manly hand on the bar
Put your hand over mine, no holding hands, we are on the run Please cop dear cop don't come with us so far we are on the road that leads to the camp one day only one half, where is the other I fled with beginning that night
The cop is far away and so is the mango tree
But the sun on the bicycle, the moon on the bicycle gallons of wind blowing our way
speeding breathlessly do I see the uniboys and the uni girls waving in the dark night studded with mango trees? My hair is in his eyes and his heart's song I hear far away the day is waiting with our future in the hand, and gates are open we are tired rattling miles in the happy land a dream was in his eyes and his heart's song I heard clear and beautiful I shouldn't fear
all the evil
chasing behind
all the hallucination
of the mind
throw the stones on the Satan's wall.
dear gods we have come, and why I mutter faintly future is waltzing in a glad glad day
100

Agate Days.
Put that day back in the silver casket someone is groaning in sleep.
One next to me is crying bitter the day is polished silver dragged out of the casket slow and reluctant hold your plate fool, don't you want your food? couldn't have got this in your shack up north that’s your sambal and these are your thosai we eat from the same plantain leaf
we arrive in his house
taste is fresh
Don't you want your food woman In my plantain leaf my rice water is in the mouth I hold my plastic plate for two spoons of rice the dhal spills over such a glum day Be quick, put back all the days and click the days together mouthful of dhal and rice and Sand
a gust of wind brings bales more the hot sand shimmering is bloody under the sun far away the tamarind tree
waved unseen hands
calls me calls me.....
I am here.
Days
Silver days are forbidden Silver days are taboo wait for the bullets and the gun fire to light them in the night
Silver days are taboo.
101

Page 56
á هلال
I am the thief
I wait to drag out another agate day count them on my beads a long long string. Where do I go for my toilet? Can I sir, have a jug of water? I don't see the soap he sees up there, I don’t see the water you see in the bushes where mines are buried I need water.
I can't lie on the floor without a mat and my man I fled with count oh count the agate days my child in my womb I can rock her for ever and a day in my warmth is her security
a man went looking in the night
for his roots and a call
to be someone
in a Sovereign State.
open the casket what a slow coach I have become
Quick the lid of mercy opens easily
Snatch that agate, it's the brightest mother said come home my child father said no come, you abandoned us
in misery and shame.
102

Agate Days
Some gloat when we drown in shame child
we are drowned.
but my agate day is Sweetest go on the sandy land
under the neem tree
resting, looking for a living place just for two with a window and door bed for sleep time sleep time is dead time when you wake up push back the misery don't bring back vanished days just the living ones
just my
agate days
linking me with him
no blue of the river
comes down my way it's not binding us
into one
anymore
In this hot camp lit by gun fire
I ShỉVer
but my agate days remain the brightest.
O3

Page 57
.á 42ره
Vivienne Goonewardene - An Official Assessment of the Lanka Sama Samaja Party*
ivienne Goonewardene was born into a man's world in the year before the Russian Revolution. It was the men who fought wars, made revolu tions and ruled the home. In her own homc her father held the view that women did not need higher education; that caste was a regulatory principle in Society; and that the family was a patriarchal institution. She had a problem of liberation in her own home.
Vivi was the niece of two uncles in the Boralugoda clan who, by the time she grew up, had attained fame as revolutionary militants, Philip and Robert Gunawardena. While still at Musaeus College she became involved in the Suriya Mal campaign which contrasted itself to British militarist traditions. Her father allowed himself to be persuaded to let Vivi attend the University College. But before long he changed his mind and kept her at home. He may have anticipated challenge to his paternal authority lurking in the atmosphere of university co-education. For his daughter had what most men appreciate in their women companions, viz. distinct good looks. He also had to be aware of the peril of revolutionary values deriving from his two brothers-inlaw. The result was that Vivi had to prepare for her degree examination by stealth with the assistance of her Uncle-Robert.
*These are the contents of the scroll presented to Ms. Vivienne Goonewardene on the official birthday celebration organised by her Party.
104

When the news of his daughter's success at the degree examination became known, the man's rage increased. Her wish to marry Leslie Goonewardene impelled him to intensify the rigours of Vivi's confinement. Only the intervention of the Supreme Court could finally establish her right to marry a man of her choice. The equality of Sexes in bourgeois Society was an abstract legal principle. But it had little relation to the reality that obtained.
Vivi was not what could be called a feminist. The attraction of her personality was not easy to resist. She was able to find hers in male society. She became an influential political figure. She had courage and fighting spirit and boundless encrgy. She could take on all comers. police, politicians, thugs. Nothing ever daunted her. She was on every LSSP platform, never at a loss for words, a great drawer of crowds, especially women. From Anuradhapura to Colombo, Ratnapura and Matara were all in a day's work. Generous to a fault, there were many who were able to exploit her charitability. But she never complained. She was kindness personified in her identification with the poor, the sick and suffering. Her political life derived motivation only partially from theory or ideology. Ideological battles drove her to furious onslaughts and acerbic vituperation against opponents. But they did not dislodge her genuine concern for people. Service to everybody who needed her was the substance of her politics. Even in the midst of severe illness she would take up the telephone or use her letterheads to find relief to numerous Suppliants at a hospital, dispenSary, government department, police lock-up, mortuary with no thought at all about herself.
It is true that in her service of the poor and sick and necdy she did not make distinction of gender. But the very fact of her presence was a source of great encouragement to women to come out and fight for their rights. And they were not disappointed. "Viviene Sahodari” was always there in street demonstrations, outside embassies, in May Day marches, at election rallies, defying the police, shouting slogans. She had to be there unless for some unusual reason, she could not.
There were no problems that were too small for Vivi to tackle. Gravely ill in intensive care in hospital, people have been known to be insensible enough to ask for letters from her regarding trivial complaints. In this way she brought the feminine quality onto the political scene in the country. We remember that particular occasion at the Kollupitiya Police Station not so long ago, when she fell on the floor after being pushed by a police officer. Without shame or sense
105

Page 58
.á الأر
of honour, the man put his foot on her, whereupon Vivi immediately asked him: "Do you have no mother?".
Congratulations, Vivi on the selfless, colourful and useful life that you have led during the greater part of 80 years
May you continue to fight for the rights of all persons, male or female, rich or poor,
for many more years to come
106

Statement issued by the Women's Education and Research Centre on the Rape and Murder of Krishanthy Kumaraswamy
man dignity in Sri Lanka, we are appalled at the recent case of the
gang rape and murder of Ms.Krishanthy Kumaraswamy who disappeared at Kaithady checkpoint on 9th September 1996, while returning home after sitting for her "A" Level Examination paper. The murder of her mother, brother and neighbour who had gone in search of her also adds to the gruesome nature of this heinous crime.
A SaWomen's Organisation concerned about social justice and hu
It is women who have always mobilised against war. While we condemn all atrocities and violence committed by both sides to our conflict, we are particularly concerned about how a woman's sexuality becomes vulnerable during times of war.
According to general principles of International Law rape during times of armed conflict is a war crime and States are expected to prevent, prosecute and punish all those who perpetrate crime against humanity. Protocol II at the Geneva Convention which relates to internal conflicts within nation States explicitly prohibits rape as a War crime.
107

Page 59
M2 %
Moreover, we are deeply disturbed at the sense of impunity initially displayed by those who committed these crimes. It is important that the armed forces and police are trained and educated to respect the human rights of the people including a woman's right to life and Security. It is essential that armed forces and police of a democratic state should be instructed to maintain the highest standards of human rights accountability. The relevant authorities must make it extremely clear that this type of behaviour will not be condoned or tolerated.
While we welcome reports that arrests have been made and that the process of justice and accountability have begun, we call upon the Sri Lankan State to take all necessary measures to ensure that the perpetrators of these heinous crimes are prosecuted and convicted.
What is said on rape by a male social scientist :-
Purushottam Agaryal *
In Surat, Savarkar and Draupadi: Legitimising Rape as a Political Weapon', Agarwal scrutinises rape as a recurring metaphor - and reality - in communal violence. Communalism, according to him, is a political project sustained by the imagination of communal subjects in whose perception each episode of confrontation between communities is actually a battle in the ever ongoing, ever unfinished war between two racially defined nations. This sort of mythologisation of communal violence, he contends, explains why normally reprehensible acts Such as rape often gain widespread ideological rationalisation, even justification. Rape comes to be regarded, in the context of communal violence, as an explicitly military act and more. It is transformed into a spectacular ritual, a ritual of victory - the defilement of the autonomous symbol of honour named woman of the enemy community in revenge - cum - defence against anticipated assaults on the same em
blem from one's own community.
* Courtesy Economic and Political Weekly. Vol.XXXII Nos.1 and 2 January 4-11, 1997
108

PRESS RELEASE VIGIL FOR KRISHANTHI
\ X 7 e are a diverse group of women, human rights workers, and other concerned individuals, coming together to seek justice for the brutal gang rape and killing of Krishanthi Kumaraswamy and Subsequent killing of her mother, brother and neigh
bour in Kaithady, Jaffna. We deplore this and similar incidents of disappearance, rape, and murder and demand action from the government.
We call upon all citizens to join us in a VIGIL to remember Krishanthi and her family and to protest against this kind of brutality against innocent civilians in this so-called "War for Peace'.
We commence this VIGIL with your participation on October 30th, 1996 at 12.30 p.m. in the Independent Square, Colombo. We will continue to remember and protest every other day at the same time, at the Hyde Park Corner until justice is done on this issue.
It would be more appropriate if you come in black or white or mourning clothes.
109

Page 60
M2 á
Statement on Rev. Tissa Balasuriya's Eccommunication
in Sri Lanka, we are deeply distressed that Father Tissa Balasuriya, a
champion of social justice and a pioneer in the fight for human rights and ethnic harmony in sri Lanka has been excommunicated by the Vatican. We are particularly disturbed because one of the issues over which this drastic and extreme action has been taken concerns his attitude and analysis towards gender equality.
A s women who have struggled for women's equality and empowerment
Father Tissa Balasuriya has been in the forefront of the struggle for human dignity in Sri Lanka and is widely respected not only by the Sri Lankan Catholic Community, but by all Sri Lankans, regardless of ethnicity, religion or class who have benefitted from his activist intervention in Sri Lankan Social and political life. Father Tissa Balasuriya has always stood for the equality of women both within the church and in Society and has spoken loudly and eloquently on women's issues. His stand on the priestly ordination of women is only a reflection of this theological conviction. It is with sadness that we realize that this same conviction that women should be ordained as priests is one of the main reasons for his excommunication.
As women interested in international human rights, we are also concerned about the lack of the process in the proceedings which have led to his
110

excommunication. Father Tissa Balasuriya was not given an opportunity to be heard, nor an opportunity to cross examine the witnesses against him. The principles of Natural Justice which form the core of international human rights, do not seem to have been respected or followed.
As women committed to equality in our society, we also request the authorities in the Vatican to openly welcome debate and dialogue aimed at securing equal rights for women within the Catholic Community and in society at large.
We urge the Vatican to reconsider its decision to excommunicate Father Tissa Balasuriya and that arrangements be made for a fair and impartial hearing of his point of view.
11th January 1997.
111

Page 61

Subscription Rates for Nivedini
North America : USS 50 UK and Europe : USS 35 India and S.Asia : USS 15 Sri Lanka : SLR 300
Subscription form
I wish to subscribe to Nivedini / I wish to exchange our journal with
Nivedini.
米米米米米米米米来水来水水米米米米米米>k米米米米米米水水米米米米米米米米米米米水水米米米米>k米米米米米米水米米米米
Name ..................................................................................
Address ................................................................................
SC C C C 0 C LC C C S S LLLLL LL LLL LLLL LL LLL CLSC SS SSLS C LL 0S CLL LLLL 0L LSL SLLSLSS SLS L SLSS 0SLLLL SL LL LS LSLL LS S 0SL LSL0S L LLLL LLLL LLS0S LS S LSLLSL 0S000 LLLL LSL LL LS LS LL LSL 0SL0SSL LSL S 0L S LLL
Date ....................................................... •••••••••••••••••••••••••••
Encloseed is a cheque/money order drawn in favour of
Women's Education and Research Centre No.58, Dharmarama Road,
Colombo 06.
Sri Lanka.
Tel : 590985/595296 Fax : 5963 13

Page 62
Niv
Niwcclini hi.: IILIl Trilingual (LIrritls Education and Research CILIC Wer: irst
The Irlectives of the VIIIrlare:
ר
4.
To circalc, s.lrcing her incl diss gcindel' globally. To gest:Lhis li {1 - 11 CLWork bri:Llyw coch planners. To act is a callyst in scictly it lili women and ht:Icc the transforTTlati To critique and reassess the proces
The Jalur fall yr Irld publisli ;
Intcrdisciplinary rcscarch in thic sic Conceptul and airhillytical pipers of |Reviews of bıçık 3. films and the:ılır Reports ("In Italicial III intcriatiLJIN ctc.on Wonicn's Studies,
Guidelives for the currillers :
Tht: Journill Wolcon1c5 3ubl'IThi85ion in : il paper/articles Wrillen from an interdisciplin'
Research papers should be of 5-2C Rewicws should bic :pproximal cly Non-sexist language: be used. Papers must bc in English. All papers/revicws etc. must bic sent Title page of the paper should bc ty luulor's name. I'll address II di Lucli An abstrict may be submitted Iol C A statenmcent of two (or hirce lines separale sheet. The Journal cart pay contributor Journal. Papcrs shelcl bc mail:xl Lu Lihc foll
Editor
Niwclini, Journal of Women's Educati II anti Researcl No, 58, Dharmilir:T Roald, Colomb (lit.
Sri Lilrıkl.
T: ; 59(1985/59.529s, Fix .59 (33

dini
inglish. Sinhala and Tamil) of Women's nublished in 1993.
:Tnin tc informatican and knowledge II
wome, cscarchers, lobbyists and policy
ge and to bring about the empower Ilcnt of I of skiicty.
of gasning Ind disseminuting knowledge.
lil kif wichrihcn's studies.
themes related to wouncil's status and Tole, ical pCrformances. al Conferences, Symposia and Workshops
disciplines and is especially interested in Ilry approach on gender issues. | N4 pages.
ol 5[]()-{{}{) words.
in typcdon A4 papers and he double spaced. pel separticly a rhed should includc the 'phone numbers. xcccding 100 words in length.
about calch : Luthor should hic typed on al
- E:ll Luthor Will Tucceive (Ile: isSLI e Colf I hic
wing address:
(Centre.
ISSN 139-7 Printed by Kiirullaratne & Sons Ltd.